Wikipedia:Reference desk/Science

From Wikipedia, the free encyclopedia

This is an old revision of this page, as edited by 71.100.1.14 (talk) at 17:19, 22 March 2008 (→‎Genetic mutation). The present address (URL) is a permanent link to this revision, which may differ significantly from the current revision.

Welcome to the science section
of the Wikipedia reference desk.
Select a section:
Want a faster answer?

Main page: Help searching Wikipedia

   

How can I get my question answered?

  • Select the section of the desk that best fits the general topic of your question (see the navigation column to the right).
  • Post your question to only one section, providing a short header that gives the topic of your question.
  • Type '~~~~' (that is, four tilde characters) at the end – this signs and dates your contribution so we know who wrote what and when.
  • Don't post personal contact information – it will be removed. Any answers will be provided here.
  • Please be as specific as possible, and include all relevant context – the usefulness of answers may depend on the context.
  • Note:
    • We don't answer (and may remove) questions that require medical diagnosis or legal advice.
    • We don't answer requests for opinions, predictions or debate.
    • We don't do your homework for you, though we'll help you past the stuck point.
    • We don't conduct original research or provide a free source of ideas, but we'll help you find information you need.



How do I answer a question?

Main page: Wikipedia:Reference desk/Guidelines

  • The best answers address the question directly, and back up facts with wikilinks and links to sources. Do not edit others' comments and do not give any medical or legal advice.
See also:


March 16

Toxicity of non-stick pan lining?

I know it's not the recommended methd of cleaning them, but I often use rough scrubbing pads on my cheap no stick pans. The coating does flake off slowly. If I am ingesting this material in small quantities, should I be worried?--68.237.2.101 (talk) 03:15, 16 March 2008 (UTC)[reply]

See Polytetrafluoroethylene. Basically if it's Teflon and hasn't been overheated it is chemically inert and should pass straight through your body. --antilivedT | C | G 03:42, 16 March 2008 (UTC)[reply]

I personally develop a bad skin condition whenever I eat something cooked in Teflon, so it can't be entirely harmless. And no overheating is not the issue. Dupont has a vested interest in having you believe their stuff is safe if used properly, so what they tell you cannot be trusted.

If I were you I'd err on the side of caution and get some regular cookware -- stainless steel, cast iron, copper if you're wealthy. But it really depends on how concerned you are about your health and whether doing a little extra scrubbing to clean normal cookware is something you'd be willing to do. If you aren't a big-time health nut it's probably not something you need to worry about. Vranak (talk) 15:20, 16 March 2008 (UTC)[reply]

This is a contentious issue, and as far as I know there isn't a definitive answer yet. As Vranek notes, the manufacturer has a vested interest, so don't believe everything you hear.
Here are the facts as I understand and observe them:
  • Teflon and related compounds are quite biologically inert; they'll pass through you like a rock.
  • "Burnt" Teflon is reasonably nasty, and is to be avoided.
  • The non-stick coating on non-stick pans does flake off. More so with cheap pans.
  • The non-stick coating is much more likely to flake off it it's been overheated, i.e. burned. Danger, Will Robinson.
  • The non-stick coating is easily damaged (and made much more prone to both burning and flaking) by the use of metal implements, either while cooking or cleaning.
My own conclusion is that the potential toxicity of burnt-and-flaked-off Teflon is suitably worrisome that it's worth avoiding.
If you want to clean your cookware so vigorously that the non-stick coating flakes off, pretty soon it isn't non-stick any more, and you might as well use non-non-stick cookware, instead. Then you don't have to worry.
Personally, I don't like non-stick cookware, because I can't be bothered to be careful of it. (Plastic spatulas are teh pitz; they melt if you leave them in.) I find that with proper cooking techniques, my ordinary (non-non-stick) cookware doesn't stick badly enough to worry about. YMMV. —Steve Summit (talk) 15:36, 16 March 2008 (UTC)[reply]
Also, steel wool is absolutely indispensible for regular cookware. Vranak (talk) 17:00, 16 March 2008 (UTC)[reply]
Who uses plastic spatulas? Silicone or wood are much better Nil Einne (talk) 07:30, 18 March 2008 (UTC)[reply]
And why would you use steel wool on a non-stick pan ? It's guaranteed to destroy it. You might as well polish your car with steel wool. StuRat (talk) 17:31, 16 March 2008 (UTC)[reply]
I said it's good for regular cookware. i.e. non-non-stick cookware. Vranak (talk) 19:33, 16 March 2008 (UTC)[reply]
Right. I wasn't replying to you; please note the indentation. Apparently the OP is also using steel wool, or something just as bad, on non-stick pans. StuRat (talk) 22:39, 16 March 2008 (UTC)[reply]
Well, a rough scrubbing pad could mean many things. What matters more than the roughness is the abrasiveness. If the grit in the scrubbing pad is too hard, it will scratch the teflon. The OP may want to invest a non-abrasive nylon scrubbing pad. Or switch to Cast-iron cookware, which has the benefit of improved heat retention. Dforest (talk) 21:44, 17 March 2008 (UTC)[reply]


(edit conflict)Hi. I've watched infomercials on TV way too often, and on one of them, there is a solution to all your problems: SmartWare. Try Googling it or something. Remember, this offer is not avalible in stores. Don't delay, order your SmartWare today! Call within the next 30 seconds and recieve my special bundt cake pan, as well as our cake stencil: Turn a ho-hum cake into a yum-yum cake! You should have at least two to three bowel movements a day. Less than one bowel movement a day is irregular, and can lead to constipation. (Dr. Ho) Hope this helps. Thanks. ~AH1(TCU) 19:39, 16 March 2008 (UTC)[reply]
Yes, it seems as if infomercials are full of Ho's selling crap. StuRat (talk) 22:41, 16 March 2008 (UTC)[reply]

Horizontal search functions (in visual perception)

Does anybody have a clue what could be meant by the expression horizontal search functions in the following sentence: "Targets that yield horizontal search functions are assumed to reflect visual primitives, the basic building blocks of perception." (PS this is a sentence in my textbook, not a sentence found in Wikipedia.) Lova Falk (talk) 15:11, 16 March 2008 (UTC)[reply]

It's just a guess, but they may refer to a heirarchy of objects, like the following example:
  Humans
  |    |
Men  Women
     |   |
   Barb  Sue
So, a horizontal search would be a search at the same hierarchy level. For example, "Is that Barb ?", "No", "Then is that Sue ?", etc. StuRat (talk) 17:26, 16 March 2008 (UTC)[reply]
This sounds very close to a classification project I was working on. In my project the term "horizontal" was used literally to suggest the conjunctive sense of "and" when a glyph was used as a modifier of another glyph. "Vertical" on the other hand was used to suggest the conjunctive sense of "or". Thus a word is considered a "horizontal" set of glyphs and a sentence is then a horizontal set of words, whereas alternate glyphs, words, phrases, and sentences are deemed "vertical".

It would help if you gave us a little more background on the textbook and the section in question. One isolated sentence from any textbook is going to be hard to make sense of without context. Give us the whole paragraph and the title of the book, at least. Otherwise people are going to be just guessing blindly. --98.217.8.46 (talk) 18:12, 16 March 2008 (UTC)[reply]

Is Energy relative?

This is perhaps a very silly question, but nevertheless, it has troubled my from time to time. What's wrong with the following line of reasoning:

  • Velocity is relative (depends on the reference frame)
  • Therefore, its time derivative, acceleration is relative
  • Force, which is acceleration times mass, is relative (or, we can say, momentum and its rate of change are relative)
  • Thus, quantities like kinetic energy (m*v^2/2) and work (force x displacement) are also dependent on reference frame.

Is this last proposition correct? deeptrivia (talk) 16:02, 16 March 2008 (UTC)[reply]

Yes, but the second and third aren't. Acceleration isn't inherently a frame-dependent thing. A speedometer can only tell you the relative speed between two objects (for example, your car and the road), but an accelerometer can tell you its own intrinsic acceleration without reference to anything else. Inertial navigation is based on this principle. -- BenRG (talk) 18:37, 16 March 2008 (UTC)[reply]
So, there is such a thing as an absolute acceleration? If so, how can we ever find it? For example, the earth itself accelerates as it revolves around the sun, and the sun might be accelerating around the center of the galaxy. Is it that these effects are small, and in principle measurable? Do the sensors we use include all these accelerations? deeptrivia (talk) 21:16, 16 March 2008 (UTC)[reply]
Yes these small accelerations are absolute and measurable. The accelerations ,that are a direct consequence of the gravitational forces in the universe, are making the changes in velocity that make astronomical bodies orbit each other, rather than move in a straight line as they would do if no force operated on them. GameKeeper (talk) 23:16, 16 March 2008 (UTC)[reply]
It's a key tenet of general relativity that accelerations are not relative except that they can't be distinguished from gravitational fields. You can use an accelerometer to measure accelerations; the most simple of which is a cup of coffee. Start to accelerate it, just a little—you'll start to spill it. But if you have it in an inertial frame, no matter how fast you are going (say, 67,000 miles an hour), it'll sit there placidly.
Einstein used to comment that theory of relativity was something of a misnomer: what's important is not what is relative, but the very few things which are not relative, like the speed of light. It is from finding the few things which are not relative—and the implications of that—that the genius of his theory comes. --98.217.8.46 (talk) 00:59, 17 March 2008 (UTC)[reply]
KE and momentum both depend on the reference frame —Preceding unsigned comment added by 79.76.144.62 (talk) 02:19, 17 March 2008 (UTC)[reply]

DBasing the money supply

Wouldn't it be possible to enter the serial numbers that are already on US currency into a database, record the serial numbers of any currency that is stolen in the DB, scan all bills whenever they are tendered at a business and run a database check, and thus instantly know when any stolen money is tendered ? The weak point in the chain seems to be getting businesses to pay for the devices, but I imagine some tax incentive could be offered to get them to agree. StuRat (talk) 17:21, 16 March 2008 (UTC)[reply]

Such a system could easily be built based not on a piece of new equipment but merely on the upload of a digital picture of a bill using a regular scanner. This would also help everyone including the Secret Service to determining which bills were counterfeit. If yours turned out to be counterfeit then of course you would be stuck with the loss unless you could spend it quickly somewhere a scanner was not in use and the system did not require your name and to turn in the bill if it was found not to be legitimate. —Preceding unsigned comment added by 71.100.174.10 (talk) 17:43, 16 March 2008 (UTC)[reply]
I don't quite understand what you mean, StuRat. What is preventing the counterfeiter from copying legitimate serial numbers onto fake bank notes? --Bowlhover 18:01, 16 March 2008 (UTC)
Serial numbers are unique and associated with other components of a bill which can not be duplicated exactly. Pictures of every bill issued already exist. Of two bills with the same serial number figuring out which one is real and which one is not is a relatively insignificant problem. —Preceding unsigned comment added by 71.100.174.10 (talk) 18:11, 16 March 2008 (UTC)[reply]
I was talking about the theft of valid money, not countefeit money. However, the system could be somewhat useful in apprehending counterfeiters, too. If, for example, somebody tries to spend a large amount of currency and several of the serial numbers are listed as being in the possession of other banks and businesses, it might look suspicious and security might want to detain the big spender until the police arrive. StuRat (talk) 18:38, 16 March 2008 (UTC)[reply]
The potential cost of maintaining and using a database and a system that would hold information for all of the billions of US bills in circulation would be prohibitive and probably outweigh whatever is lost in counterfeiting. The labor required to enter in every bill one received and, presumably, all of the bills that one gave out (how else would you know which ones were stolen?) would be unreasonably large. Is it possible? Yes, sure. But wildly impractical. There's no way the gains would be worth the costs. --98.217.8.46 (talk) 18:02, 16 March 2008 (UTC)[reply]
And just to illustrate: say I run a business, even one that deals with a relatively small amount of money, like a convenience store. Let's pretend I have an automated machine that'll scan all of the money I put into the till, and registers it in some sort of giganormous Treasury database which always works (despite store millions of new transaction entries a day from around the country) and is miraculously fast enough to not make every bill check take as long as a credit card takes to process (about 10 seconds a card swipe or so, so if you paid for a $5 item in ones then you've got to sit around for a minute). (And note: yes, the business WOULD need to check with the centralized database every time they received a bill for change, because they too are also in the process of looking for "stolen" money from other businesses, right?) OK—best case scenario so far?
Now a customer comes, and I have to take the money they give me, scan it, and then take the money I give them, and scan it. Because I don't keep track of what I've taken out of the till, when my store gets robbed, I won't be able to distinguish between "valid" money that I gave out as a change and "stolen" money. Let's even pretend that this was somehow practical, and the system says that a mile away, someone put one of my five dollar bills into a till at a grocery store. The police are called, the suspect kindly waits for them to arrive, and everybody has a nice conversation about where that five dollar bill came from. Maybe, in an ideal world, the suspect matches a perfect description from the robbed cashier, they put him in a line-up, they identify him, they convict him. Bravo. But what if he doesn't match? What if, say, he got the five dollar bill from someone else? OK, now we've got a complicated situation here—maybe we try to shake him down and find out where he got that bill from. And so on and so on. The question is: how many times do you have to "catch" the wrong person for 1. people to think this is a waste of time, 2. businesses to think this is a waste of resources, 3. law enforcement to think this is a waste of resources? Not many. --98.217.8.46 (talk) 18:07, 16 March 2008 (UTC)[reply]
Okay you want special equipment. NCR can build a cash register with a bill scanner inside above the drawer. You put the bill in the drawer and the cash register does the rest. As for a waste of time... such a system has the potential to waste a crook's time rather than the police by simply going the extra step of requiring validation of the bill before it can be spent. Bill gets stolen the system is notified by pressing the "theft" key and poof the bill is no longer valid until re-certified by the police. —Preceding unsigned comment added by 71.100.174.10 (talk) 18:16, 16 March 2008 (UTC)[reply]
There is the voluntary project, http://wheresgeorge.com -- kainaw 18:17, 16 March 2008 (UTC)[reply]
Too many holes to be worth the effort. —Preceding unsigned comment added by 71.100.174.10 (talk) 18:23, 16 March 2008 (UTC)[reply]
Still a ridiculous suggestion. Do you think the police want to spend their time becoming "bill re-certifiers"? No law enforcement agency in the world is going to want to take on that task. The best way to catch criminals involves talking to other people, looking at specific forensic, not trying to track down each and every physical dollar. --98.217.8.46 (talk) 20:28, 16 March 2008 (UTC)[reply]
Thanks for confirming everyone's concept of the police as worthless lazy donut eating taxpayer supported freeloaders and do nothings until the opportunity arises to write a parking ticket or a ticket for driving near enough to the speed limit that in their minds it might as well be speeding. Not to mention being self-righteous oppressors of Berkley student freedoms. —Preceding unsigned comment added by 71.100.174.10 (talk) 20:57, 16 March 2008 (UTC)[reply]
I tend to think of police that way myself, as the only interaction I seem to ever have with them is when they extort money from me via tickets. Since I am neither a pretty woman good at flirting nor another policeman or government official immune to traffic laws, I never get out of a ticket, either. When anyone is driving like a fool and actually endangering lives, no police are ever to be found. I even saw one instance where police apparently put up a barracade and stopped traffic with the sole purpose of ticketing those who drove around it. StuRat (talk) 22:30, 16 March 2008 (UTC)[reply]
Many of the limitations listed are only current limitations, which can hopefully be solved soon. I've never understood why credit card approval takes so long, and increasing bandwidth, etc., should make this process quicker soon. Also, rather than "dialing in" for each bill, something like a DSL system can be always on and ready for a check. Looking up a unique serial number in a database is one of the quickest DB operations there is. Initially, perhaps only banks would record the location and distibution of currency, with businesses merely running a check, not updating the database. A few years later on, businesses could update the records, too. Note that the full system could also be used as an anti-terrorism tool, as terrorists often need to move large sums of money around. If they go to a bank and withdraw large sums of money, the serial numbers could be tracked and we would know, say, if they used it to buy weapons and explosives. It would also be interesting to know if large sums of money given to Islamic charities are being used to purchase weapons. StuRat (talk) 18:50, 16 March 2008 (UTC)[reply]
The whole thing is totally impractical with paper money. And at least in the United States, no arms dealers are going to start using such a system—a lot of the transactions would be "off the grid", especially for those involving potentially scandalous or illegal activities. There are better ways to check how businesses and charities spend their money if one is inclined to do so.
It doesn't matter whether reading the index of a bill is a quick operation—even a relatively quick operation needs to be able to scale. And every complex system is going to have major downtimes, bugs, etc. in it.
My money (haw haw) would be on using electronic money and getting rid of paper money long before any of this would ever become practical, and I still think that even if you did have a system of tracking paper money it would still be wildly impractical. Certainly more impractical than just making everyone switch to purely electronic currency and skipping the paper step altogether. If countries make universal IDs mandatory (which I suspect they will in the next decade or so), that would make it all the easier. --98.217.8.46 (talk) 20:24, 16 March 2008 (UTC)[reply]
Certain "high-risk" businesses, like gun shops, pawn shops, etc., could be forced to comply by law. It would be easy enough to verify, just have an undercover cop go in with a "bad bill" occasionally, and, if they don't report it and refuse to sell the gun, give them a whopping huge fine. This method would allow the police to make a nice profit, ensuring that they would actually enforce the law. StuRat (talk) 22:21, 16 March 2008 (UTC)[reply]
(And if you're worried about people buying guns, why not make them register the guns? There are far less guns than dollars, and they're much larger, much more worth the effort. If you think people don't want you to track their gun-buying habits, or that they can find ways around that check, why would you assume money would be any easier?) --98.217.8.46 (talk) 20:30, 16 March 2008 (UTC)[reply]
People don't usually leave guns around after a crime. And, if they want to get around the tracking, it's a lot easier to remove a serial number from a gun than from every bill you ever have. StuRat (talk) 22:12, 16 March 2008 (UTC)[reply]
What has been learned from previous experiments to track money is that criminals create their own from of money and find other ways to thwart tracking like through the construction, purchase and sale of real estate. A "company" buys a lot for twice what it is worth, builds on it and sells the lot and the building for eight times the going price of any adjacent property. People are so open to a free market under capitalism they do not even bat an eye. Money gets laundered like this around here all of the time and no even cares.
This might help track down the laundering operations, too. If an armored car is robbed, then, a year later, somebody buys a $100,000 home for $200,000 in cash, and many of the bills are those stolen from the armored car, the buyer would have some explaining to do. StuRat (talk) 22:15, 16 March 2008 (UTC)[reply]
Why would you assume that they'd be using that money anyway? If a ridiculous tracking system went into effect, what's to prevent someone going outside of the country to launder it? You severely overestimate the technical abilities needed to do such a thing while at the same time you underestimate how easy it would be to get outside of the grid. Again, there's nothing your system would offer that a generalized electronic money system would not, except that yours would cost billions more to produce and maintain in both infrastructure costs and operation time, and because you still were trying to rely on physical money you make it exceptionally easy to evade your proposed system. In any case, armored cars are not robbed all that often and the companies that run them are insured anyway (you're using a very clunky hammer to try and hit a pin of a problem). --98.217.8.46 (talk) 00:55, 17 March 2008 (UTC)[reply]
Moving currency offshore in large quantities has it's risks, too, as anyone repeatedly smuggling large sums of cash is likely to be caught. The armored car was just an example, it doesn't really matter where the currency was obtained illegally. And, whether they are insured or not is quite irrelevant, it only means the insurance company now wants the cash recovered instead of the original target. StuRat (talk) 01:07, 17 March 2008 (UTC)[reply]

Interesting question, StuRat, but lets take the suggestion even a step further:

  • Instead of carrying paper currency around, why don't we store the serial numbers of the currency that each person owns in a (verifiable and secure) database.
  • Then instead of presenting the paper bills to (say) a gun shop, one only needs to provide a secure ID and the database is remotely updated to transfer ownership records of the requisite amount of paper currency.
  • As a last step, clearly we don't need to store the actual "serial numbers" of the notional "paper" money, but only the amount of money that each person possesses at any particular point of time.

Looked upon this way, your proposed system is just an insufficiently ambitious version of an all electronic financial system, which (in developed countries at least) accounts for (guesstimate) 99.9...% of all financial transactions (once we account for all financial activities, i.e., not only retail consumer purchases).
Aside: We may well be living in the the last-few-decades of the physical currency era, and future generations may look back with amusement and disbelief at our primitiveness, just as we consider a barter economy obsolete and limited.
PS: The only relevant wikipedia article I could find is the one on electronic commerce but the scope of that is too narrow for our discussion. Abecedare (talk) 02:22, 17 March 2008 (UTC)[reply]

I don't agree with 99.9% of all transactions being electronic. Perhaps if measured by percentage of total transaction value, that would be correct, but many small transactions, like fast food restaurants, paying the paper boy, etc., still are largely done with cash. In a society without cash, bums would have a hard time getting a hand-out, too (I picture them carrying a credit card scanner to accept donations :-)). I, for one, use cash whenever possible, as there is no possibility of ID theft when I hand the cashier cash (and if a cashier doesn't take cash, do they become a "creditier" ?). StuRat (talk) 05:31, 17 March 2008 (UTC)[reply]
  • Yes, I meant 99.9...% by value (I'd be curious to know the true stats).
  • The bum wouldn't have to carry credit card scanners. All you'll need to do is note down their Database ID number (perhaps, their ID card will communicate wirelessly with your "mobile phone") and transfer funds electronically to their account, which they can then access instantaneously. Note that this perfectly implementable using today's technology, although clearly there are sociological, economic, ID theft and other non-technological issues. IIRC, on several university campuses one can pay petty expenses, such as vending machines, laundry etc using the student ID card and that may be a first step towards a cashless society with all its benefits and ills.
Cheers. Abecedare (talk) 06:01, 17 March 2008 (UTC)[reply]
An interesting effect of a cashless society would be that it might force many back to a barter economy. An illegal immigrant likely can't use a credit card, so, without cash, they would need to be paid in room and board and maybe pesos, if they weren't also eliminated. StuRat (talk) 19:31, 17 March 2008 (UTC)[reply]
Again hardened criminals and just ordinary people who want to avoid paying income and sales taxes barter all of the time without cash or debit cards getting involved. You sell a house to someone for $950,000 no one is going to require you to show where they actually paid or where you actually deposited the cash. Instead the house can be a payment in reality rather than a sale. You can avoid the grid without going offshore and by staying right here. Everyone knows how to work the system and how the system works. —Preceding unsigned comment added by 71.100.174.10 (talk) 09:30, 17 March 2008 (UTC)[reply]

artificially macerated food

As an informal experiment I have been macerating my meals using a food processor. The consistency of maceration is about the same as bean dip used for tacos but includes meats, vegetables and most components of a regular meal including condiments. I consume a beverage with the macerated meal. Not only do I become full with only half a meal but have been able to reduce the number of meals to one per day with occasional in between unmacerated light snacks. Not only do I seem to have more stamina for excercise but the pounds are beginning to come off. Where can I find scientific or medical studies which discuss artificially macerated food? —Preceding unsigned comment added by 71.100.174.10 (talk) 17:34, 16 March 2008 (UTC)[reply]

The only way I can see that helping is if the food is less palatble in that form, so you eat less of it. StuRat (talk) 18:54, 16 March 2008 (UTC)[reply]
It is somewhat less palatable but not to a degree that it is something one can not get use to in light of other benefits. What I need is a list of studies which have explored the question in depth. —Preceding unsigned comment added by 71.100.174.10 (talk) 21:14, 16 March 2008 (UTC)[reply]
I'm not aware of any studies. But what exactly would be the mechanism by which you would desire to eat less, if not that the food is unpalatable ? I suppose that might still be better than diet/weight loss methods like Alli, which apparently causes weight loss because people who take it crap their pants unless they stop eating fat. Talk about unpalatable ! StuRat (talk) 22:03, 16 March 2008 (UTC)[reply]
...you know perhaps NASA has a study. I just thought of that. Even Stanley Kubrick included it (pre-macerated food) in his film. —Preceding unsigned comment added by 71.100.174.10 (talk) 09:13, 17 March 2008 (UTC)[reply]
Check with aged-care studies, where macerated food (often reset in more attractive moulds) is used.Polypipe Wrangler (talk) —Preceding comment was added at 23:34, 16 March 2008 (UTC)[reply]
This question sounds strikingly familiar. --The Fat Man Who Never Came Back (talk) 23:58, 16 March 2008 (UTC):-)[reply]
Weathermen are notorious for earning good livings by giving different answers to the same questions every day. —Preceding unsigned comment added by 71.100.174.10 (talk) 08:55, 17 March 2008 (UTC)[reply]

How much human evolution is yet to come?

Is there any evidence regarding how close human evolution is to reaching an end state where it will stay? How much improvement is left to be made over the next few million years, assuming the planet remains habitable that long? NeonMerlin 18:22, 16 March 2008 (UTC)[reply]

That's easy. Whatever will make the human physically and mentally more efficient. An adaptation to the ever increasing abundance of pornography, for instance, might be a third hand growing out of one's thigh. ;D —Preceding unsigned comment added by 71.100.174.10 (talk) 18:25, 16 March 2008 (UTC)[reply]
As long as the environment changes, evolution will cause change in the species that survive. Even if the environment on Earth becomes static (which is not likely), humans are trying to go to other planets, which will lead to evolution to match the foreign environment. -- kainaw 18:41, 16 March 2008 (UTC)[reply]
Unfortunately, human evolution is likely to take a nasty turn, where those who irresponsibly produce the most children and abandon them are likely to pass on the most genes. Historically, the abandoned children would die, but in modern society, they are taken care of by the state. In the long run, however, the state will no longer be able to care for the expanding pool of abandoned children, and they will no longer survive. Or, perhaps before we get to that point, some fairly radical solutions, like forced sterilization, will become acceptable. StuRat (talk) 19:00, 16 March 2008 (UTC)[reply]
By chance, Stu, have you seen Idiocracy? Someguy1221 (talk) 20:36, 16 March 2008 (UTC)[reply]
No, I haven't, since doing so won't tend to increase the number of times I reproduce. :-) StuRat (talk) 21:53, 16 March 2008 (UTC)[reply]
And spending time on Wikipedia does? o_O Someguy1221 (talk) 23:38, 16 March 2008 (UTC)[reply]
Irresponsibility is not a genetic trait. There's no reason to assume those particular children are going to be any stupider than others. The complaint about "abandoned children" (or the mentally disabled, or the physically disabled) overburdening the state has been a nice scare fantasy of eugenicists for over 100 years now; the state seems to be doing just fine in that respect. It is pretty unlikely that compulsory sterilization will ever become in vogue again in Western countries. --98.217.8.46 (talk) 20:20, 16 March 2008 (UTC)[reply]
I don't agree that irresponsibility isn't a genetic trait (the Kennedy family may make my point for me). But, even if we say it isn't, children who are abandoned are also less likely to grow up to be responsible parents due to the lack of a good role model. As for abandoned children being cared for "just fine", this certainly isn't the case. In Romania, for example, they have a high fatality rate in underfunded and often abusive state-run institutions. The kids are often misdiagnosed as having severe mental and physical problems, which, while not true initially, becomes a self-fulfilling prophecy the longer they remain there. For example, since they are judged incapable of learning, they aren't given access to education, which, in turn, leaves them illiterate. The story isn't much better in the US, where children are often shuffled from foster home to home, some of which are abusive. Also, 100 years is an insignificant amount of time on the scale of human evolution, so I'm not surprised that the negative aspects haven't yet become overwhelmingly apparent. StuRat (talk) 21:47, 16 March 2008 (UTC)[reply]
I say "just fine" from the standpoint of the species and the standpoint of the survival of the state. This is a conversation about evolution, right? Abandoned children make up an insignificant part of the human gene pool, as does all of Romania. Even the United States doesn't stand for a whole lot on a biological level. If you want to talk about biology, talk about biology. If you want to talk about society, okay, talk about society. But don't pretend you are talking about one thing when you're talking about the other. For all of your talk about the effects of modern society on the long-term prospects of the human species, your views are tailored to very small issues and completely absent are any considerations that would take into account, say, the most populous societies on the planet. Don't wrap your ideology up in scientific terms and pretend it is science—that's another bad legacy of the eugenicists for you to avoid. --98.217.8.46 (talk) 00:51, 17 March 2008 (UTC)[reply]
When did I say Romania and the US were the only countries having trouble caring for abandoned children ? It's a problem in many countries. If you want to wait until we get to the point where nearly all abandoned children die to see it as a problem, then it will be too late to avoid the worst consequences by then. StuRat (talk) 01:22, 17 March 2008 (UTC)[reply]
You might be interested in the higher evolution.--Shantavira|feed me 19:23, 16 March 2008 (UTC)[reply]
I won't argue StuRat on intellectual grounds but I may say that his view strikes me as a touch pessimistic.
As to the original question, a quote from Hagakure:
A certain swordsman in his declining years said the following:
In one's life, there are levels in the pursuit of study. In the lowest level, a person studies but nothing comes of it, and he feels that both he and others are unskillful. At this point he is worthless. In the middle level he is still useless but is aware of his own insufficiencies and can also see the insufficiencies of others. In a higher level he has pride concerning his own ability, rejoices in praise from others, and laments the lack of ability in his fellows. This man has worth. In the highest level a man has the look of knowing nothing.
These are the levels in general. But there is one transcending level, and this is the most excellent of all. This person is aware of the endlessness of entering deeply into a certain Way arid never thinks of himself as having finished. He truly knows his own insufficiencies and never in his whole life thinks that he has succeeded. He has no thoughts of pride but with self-abasement knows the Way to the end. It is said that Master Yagyu once remarked, "I do not know the way to defeat others, but the way to defeat myself.
Throughout your life advance daily, becoming more skillful than yesterday, more skillful than today. This is never-ending.
I believe this to be true at the level of the human race as well as the individual. So, to answer the question with this in mind: there is no end-point in human evolution. Vranak (talk) 19:31, 16 March 2008 (UTC)[reply]
The question as to whether humans will keep evolving in a meaningful way (that is, having more than just incidental circulation of gene frequencies, etc.) and whether that will be for the better or the worst has been asked since the 19th-century. There isn't really a single right answer for it. Human selective pressures have been relatively weak on average for a long time now—we don't tend to let people die off who might otherwise, and no individuals tend to have trouble reproducing if that's what they want to do. Human genetics and human society are both sufficiently complex that the Social Darwinist beliefs that those with money/power/affluence must necessarily be genetically "superior" to those without them have long been shown to be very flawed. There is also the big question of human genetic engineering, which could lead into a variety of different possible directions. Lee Silver's book Remaking Eden suggests a number of interesting possibilities in it without making terribly large assumptions about the scientific advances needed. --98.217.8.46 (talk) 20:20, 16 March 2008 (UTC)[reply]
Assuming machines keep on tending to make life easier and humans do not suddenly come under some sort of drastic survival pressure, then any changes that occur will not be considered improvements by most of us. For instance, lack of toes will not instantly result in death or the inability to reproduce. It might have been so once upon a time, but not now. What is going to stop evolution getting rid of toes if they have no survival benefit? Or take our precious brains, very expensive in energy to maintain, but no survival value now that we have machines to do all our thinking for us. Probably first thing to go if evolution needs to save on energy for any reason. We will only be saved from this terrible future if survival becomes more difficult, or cosmetic genetic engineering becomes legal and cheap. SpinningSpark 20:33, 16 March 2008 (UTC)[reply]
If there is no selection for a trait then it's not going to propagate. And I don't know about you but my machines haven't learned to do any thinking for us; so far, all predictions of human intelligence being useless have proved pretty baseless. At the moment, for all of your fears of brains being cosmetic, most of our top paid professions require big brains, ergo the difficulty of getting into and succeeding in law school, business school, medical school, etc. If anything has characterized human society in the last hundred years it has been the increased importance of expertise and the slow shuffling towards something of a meritocracy (as opposed to the centuries of hard and fast rule by aristocracy). I wouldn't be all that worried.
As for toes, if there is no strong selective pressure for it, it won't catch on. You might think there is not selective pressure against it, but I'm betting that most toe-less individuals would probably disagree to some extent. And in any case, sometimes there is a benefit to missing a few. ;-)
Evolution and genetics is much more complicated than the "toes" example, in any case. Evolution in the face of heavy selective pressures can be quick in small populations, but in a very large population with a lack of selective pressures I think you're going to find mostly a regression towards the mean on the whole, e.g. not much change in either direction over the course of the population as a whole. (Ironically, that is what the founder of eugenics, Francis Galton, thought too. He thought this was stagnation. I consider it just to be stability.) --98.217.8.46 (talk) 20:41, 16 March 2008 (UTC)[reply]
Humans and the demands of their environments are quite mobile. Adaptation often takes the form of simply moving to a place where your deficiencies become meaningless and your talents are irreplaceable. Hence, Lovette goes to jail, Hilary and Barack hit the campaign trail and McCain heads for Iraq. —Preceding unsigned comment added by 71.100.174.10 (talk) 21:27, 16 March 2008 (UTC)[reply]
Yes it's true that brains are still valued in law, science and medicine etc. but my point is that this makes no difference to an individuals abililty to procreate. Hence, it is irrelevant to evolution. The unemployed make just as many children as brain surgeons, possibly more. SpinningSpark 21:45, 16 March 2008 (UTC)[reply]
Money appears to be the deciding factor when it comes to procreation even for the lowest class rather than brains unless brains translate into money. —Preceding unsigned comment added by 71.100.174.10 (talk) 21:56, 16 March 2008 (UTC)[reply]

Biological evolution is an ever ongoing incessant process. It occurred in the past, is occurring at the present, and will continue to do so in the future as long as life exists on the planet. Wisdom89 (T / C) 22:22, 16 March 2008 (UTC)[reply]

The only end state for evolution of humanity will be extinction or immortality. If you agree with the Doomsday argument the former may be quite soon. GameKeeper (talk) 23:28, 16 March 2008 (UTC)[reply]

Extinction is the only eventual possibility, the question is when and why. Personally I'm of the "we're never going to get off of this hunk of rock" school of thought, and would be highly surprised if we made it through the next century, judging by how the last one went. --Fastfission (talk) 23:38, 16 March 2008 (UTC)[reply]

One recent study found that human evolution had recently accelerated (though "recently here" is from 50,000 BC to 10,000 BC or so). 140.247.254.4 (talk) 01:03, 17 March 2008 (UTC)[reply]

We could also be in a state of de-evolution now that computers are handling so many of our former jobs.
Assuming a distant stable future and an extreme mastery of technology and genetic manipulations by the descendents of humans, we can imagine a world where the idea of human evolution as become obsolete. An almost total ability to genetically engineer and transform organisms breaks the barriers between what we call robots and living organism. The notion of 'humanity' has lost its meaning as it has become only a narrow range of genetic possibilities in a large spectrum of possible configurations that are all part of society, or maybe the term's meaning has broaden to include all sentient beings. So the idea of human evolution will disappear in time to be replaced by genetic engineering (and maybe we'll all look like manga and disney characters?). Keria (talk) 13:30, 17 March 2008 (UTC)[reply]
It bears mention that we haven't really been around as a species for all that long. The numbers are hard to imagine as they are -- 2.5 Million Years for humans and 3.8 billion years for the cooled planet -- but consider the first land masses on earth as having formed around midnight of January 1, and we are approaching midnight of Dec 31 a year later. That puts humans on the planet for the past 4 ½ hrs -- during the first two hours of which we seemed to coexist quite peacefully with the Neandertal, and possibly others. All of recorded history has taken place in the past 2 minutes. (The dinosaurs died out on Christmas Eve). The jellyfish, in contrast, have been around for a couple of months, relatively unchanged. In the face of these kind of numbers, it's easy to see where the question gets murky. If we are to quantifiably evolve, it's going to take tens of thousands of years, just like every other organism on the planet, which means that we've got to survive long enough to do it. And if we're talking about a speciation event, then by definition "we" will no longer be...Vance.mcpherson (talk) 21:42, 17 March 2008 (UTC)[reply]
It also bears mention that evolution of the traditional sort is likely soon to be made insignificant in copmarison to artificial, intentional tinkering. —Tamfang (talk) 23:21, 19 March 2008 (UTC)[reply]
Technological singularity seems to have a great chance to occur a lot sooner than any detectable biological evolution. --V. Szabolcs (talk) 14:30, 22 March 2008 (UTC)[reply]

Winter Solstice

In the year 1990, did the winter solstice fall on December 22nd? Or the 21st? 71.174.25.42 (talk)Winter

The winter solstice falls on the 21st during leap years and the 22nd during regular years. 1990 was not a leap year. Therefore, December 22 was the winter solstice. --Bowlhover 21:40, 16 March 2008 (UTC)
Unfortunately, it's not quite that simple: "Depending on the shift of the calendar, the event of the Winter solstice occurs some time between December 20 and December 23 each year in the Northern hemisphere." See Winter Solstice#Date, especially the graph on the left. You also need to keep in mind that the solstice proper is a single moment, which happens simultaneously in all time zones. Depending on where on the planet you are, it may be late in the evening on one day, or early in the morning on the next. BTW, [1] (found through Google) gives the 1990 winter solstice happening on Dec 22 at 03:10 UTC. Note that this is actually 9:10 pm on Dec 21 in Chicago. (Also note that the Winter Solstice happens in June for those people south of the equator, due to the inversion of the seasons:) -- 128.104.112.85 (talk) 22:49, 16 March 2008 (UTC)[reply]
For some reason, I thought the winter solstice always fell on December 22 UTC time or December 21 UTC time. Of course, that's not true; the winter solstice article states that it occurs on December 20 this year and December 21 for a few years afterwards. --Bowlhover 04:22, 17 March 2008 (UTC)

Radiography in Thailand?

Does anyone have any information on working as a radiographer in Thailand? I am studying it at the moment in Scotland and would quite like to do an elective placement there. 172.142.123.129 (talk) 23:02, 16 March 2008 (UTC)[reply]

I had a look, and curiously we don't have a Healthcare in Thailand article. Perhaps the best place to start would be the Thai Embassy in London which has a list of hospitals in Bangkok. Sorry, I couldn't find out if there was a consulate in Edinburgh. Astronaut (talk) 04:21, 18 March 2008 (UTC)[reply]

what is the most effective toothpaste?

which brand of toothpaste is the most effective? —Preceding unsigned comment added by 79.122.42.52 (talk) 23:40, 16 March 2008 (UTC)[reply]

The brand you actually use. —Preceding unsigned comment added by 71.100.174.10 (talk) 23:44, 16 March 2008 (UTC)[reply]
That would depend on what you want it to do. There is cavity prevention, tartar control, tooth whitening, fresh breath, etc. Depending on what particular problem(s) you have, a different formulation may help. StuRat (talk) 23:53, 16 March 2008 (UTC)[reply]
The one that claimed to be able to split water molecules during brushing in order to release 'activated oxygen' into the mouth for a deeper clean sounded pretty interesting. Shame they stopped advertising it... --Kurt Shaped Box (talk) 23:58, 16 March 2008 (UTC)[reply]
All you need to do that is brush with hydrogen peroxide. —Preceding unsigned comment added by 71.100.174.10 (talk) 00:02, 17 March 2008 (UTC)[reply]

I'm not sure what brands are available in Hungary, but you might want to check out an independent product reviewer like ConsumerReports.org, or even a website like Epinions.--The Fat Man Who Never Came Back (talk) 23:54, 16 March 2008 (UTC)[reply]

All of the same major ones are available here as the US. Which is the best? (Did you read independent reviews?)

As you haven't said exactly what you mean by "most effective", you might do well to read the Consumer Reports' article on various aspects of toothpastes and the claims made about them here [2]. ៛ Bielle (talk) 01:35, 17 March 2008 (UTC)[reply]
I had a small travel pack of Colgate sensitive (clinically proven sensitivity relief) promising all kinds of protection and relief from pain, which I used to clean the grunge off the baked enamel stove top -- surprisingly effective. No need for harsh scrubbiing. I'm still waiting to hear from the cooker about its new found confidence, but I'm keeping it handy for next time when nothing else works. Julia Rossi (talk) 05:23, 17 March 2008 (UTC)[reply]

please remember that the most effective toothpaste always has most number of active chemicals which means you may.. may..and not definitely.. damage your teeth. So, I always prefer the least effective from a trusted company. -Balaji —Preceding unsigned comment added by 59.92.126.151 (talk) 13:58, 18 March 2008 (UTC)[reply]

Speed in space(not the drug)

I was discussing with a friend about the age of the universe. A subject way over my head. I brought up that a quasar had been found that was 28 billion light years away from us and gaining. Yet the universe is only 13.73 billion years old. So that would mean that the quasar is traveling faster than the speed of light. It has moved farther than light could in that time. Disance in time = speed.(I know that we are also moving, so some math would have to be done) That is how I and the traffic cops understand speed. But he said that it realy was not moving that fast. The space in between us was actualy growing. My response was Duh, you don't say. That is what distance is. Anyway, he was serrious. He said I would have to understand general reletivity to understand that objects in space are not moving at speed but that space was growing. Can you tell me if he is an idiot or not? And if he is not can you explain this in common language? thanks.

cris —Preceding unsigned comment added by 68.209.69.103 (talk) 23:59, 16 March 2008 (UTC)[reply]

Metric expansion of space, yes, space is in fact getting bigger. A common analogy used in popular science it to imagine a universe that is confined to the surface of a balloon. The balloon obviously has a volume to it, but everything, including light, is permanently stuck to travelling along the surface. If you draw two dots on the balloon to represent, say, you and a quasar, and then you blow up the balloon, you two will get farther and farther apart. Each of you can look around and accurately say, "Everything around me is moving away from me, and I seem to be standing still." So who's right, and who's moving? Or is it merely space that's changing? I'm not a general relativity expert, so that's as close as I'm going to get to explaining this to you. Someguy1221 (talk) 00:08, 17 March 2008 (UTC)[reply]

OK. I understand the baloon analogy. But the two objects are still gaining distance from one another. Wouldn't that still asign speed to them? Are all objects in space spreading apart? Are they doing this equaly or are some moving faster away than others? —Preceding unsigned comment added by Loach (talkcontribs) 01:18, 17 March 2008 (UTC)[reply]

I removed the spaces from before your answer, because they put your text in an unformatted grey box ...
like this.
If you want to indent text, use colons (these things - :::::) instead. Now, on to your question. Yes you can assign a speed to the separation between two objects, but there are two caveats: (1) you can only say that they're moving relative to each other, and (2) you do so based on the frame of reference you are observing from. The "relativity" in Einstein's theory of relativity refers to the fact that as long as you keep track of things properly, no frame of reference is better than another - and most importantly, there's no "true" frame of reference, so you can't just say "that object is moving at this velocity" unless you state with respect to what. As to the rest of your questions, yes most of the observable universe appears to be expanding at a fairly uniform rate, with differences due mostly to gravitational effects (if you and I are standing on the balloon, and holding on to each other, then we won't be pulled apart as much as the balloon expands). In fact, the uniform expansion of space is what allows us to equate how far away things are with how fast they're moving away from us, proportional to Hubble's constant. And that linear relationship means that, yes, the furthest objects appear to recede faster than the speed of light. Confusing Manifestation(Say hi!) 03:03, 17 March 2008 (UTC)[reply]
There are no inertial reference frames at cosmological scales; the symmetry is broken by the overall curvature. Length contraction, time dilation and the relativity of simultaneity are meaningless in cosmology. There is a privileged time coordinate (cosmological time, the distance to the south pole on the globe below) and a privileged spatial distance (comoving distance, the difference in longitude on the globe), which can be used to define a privileged recession velocity between two objects. Hubble's law says that the (privileged) recession velocity is roughly proportional to the (privileged) distance. There is no privileged position or absolute velocity, though (i.e. no center to the expansion). -- BenRG (talk) 21:36, 17 March 2008 (UTC)[reply]

OK. Sorry for the grey box. I do not know how I did that. One last question. Does the universe have a central starting point from which the BB started? I've heard both yes and no. Do we know where it is? And could a reference on speed be made from there. thank you for your help. —Preceding unsigned comment added by 68.209.69.103 (talk) 03:29, 17 March 2008 (UTC)[reply]

You're probably hitting TAB or something before typing your questions. Anyway, there was no central starting point. If you were to run the universe backwards, and watch it with a camera in any inertial reference frame you please, you would see the density of the universe climb higher and higher, approaching infinity at the big bang. Once again, this would happen no matter where your camera was or what reference frame your camera was in. So every position in space has equal claim to having been the "central starting place." And depending on what the shape of the universe is, the universe may well have always had infinite volume and no boundaries; so it may actually be geometrically impossible to define a center of the universe. Someguy1221 (talk) 03:39, 17 March 2008 (UTC)[reply]
See my response above—no inertial reference frames. Also, it doesn't make sense to say that something is in a particular inertial frame, since every inertial frame assigns coordinates to every object. Also also, the universe doesn't look the same at different speeds (it makes a difference whether you're moving with respect to the CMBR). But it's true that the camera will see the density go to infinity regardless of its state of motion. -- BenRG (talk) 21:36, 17 March 2008 (UTC)[reply]
It's not just that objects are moving (which they are); the very fabric of space is expanding as well. Imagine two ants on a blanket, walking away from each other. But the blanket itself is also stretching wider at the same time, so the net distance that they are from each other increases very quickly. What's impressive is that this particular effect—the expansion of space—is pretty much the only way that something can end up going faster than the speed of light relative to anything else. The objects themselves are not moving faster than the speed of light, but the distance between them is growing at a rate that they end up distances apart that would require going faster than light to actually have done had the space not been expanding. Crazy, no? --98.217.8.46 (talk) 14:18, 17 March 2008 (UTC)[reply]
How can we see something farther than n light years away when the universe is only n years old? That's easy: it's simply not true that light travels a distance ct in a time t. It's never been true, thousands of confident assertions in books notwithstanding.
It's easiest to visualize what's going on in the case of a closed universe which expands, reaches a maximum size, and then recollapses. For simplicity suppose there's just one spatial dimension, which is all we need for this example. You can imagine this universe as the surface of a globe, with the south pole being the big bang, the equator being the time of maximum size, and the north pole being the big crunch. (Note that this is different from the "expanding balloon" analogy—this is a fixed-size balloon which shows the whole history of the universe at once.) We can draw lines of latitude (x axes, purple) and lines of longitude (t axes, cyan) on this globe. We'll draw a very large number of them so that the whole globe is covered fairly densely with intersecting latitude and longitude lines. What "light travels at a constant speed" really means is that light crosses all of these intersecting axes at the same angle. Usually the units are chosen so that the angle is 45°. A curve with this property is known as a loxodrome, and an example is shown in white in the image to the right. Unfortunately the angle here is nowhere near 45°, but try to imagine a similar curve with a 45° angle. Say that the present era is 30° from the south pole. The spatial size of the universe in this era is 2πR sin 30° = πR, and the elapsed time since the big bang is πR/6, where R is the radius of the globe. The first question is: how far away in this era can two objects be that started out in the same place at the big bang, and that have moved slower than light since then? The answer is obviously "arbitrarily far". In particular, they can be on opposite sides of the universe (πR/2 light years apart) even though it has only been πR/6 years since the big bang. The second question is: how far away is the most distant object we can see? The answer is again "arbitrarily far", because the loxodrome that represents the path of a light beam has circled the entire universe in the time since the big bang. In fact it has circled it infinitely many times, so we will see infinitely many images of everything in the universe at progressively earlier stages in their evolution. Note that the distances involved here have nothing to do with the speed c—they're a consequence of the large-scale geometry. The distance ct, where t is the time since the big bang, is geometrically meaningless. In the real world, the current distance to the south pole (along a line of longitude) is 13.7 billion years, and the current distance to the most distant known quasar (along a line of latitude) is 28 billion light years. Nothing in the geometry of the real world relates the latter to c times the former.
The globe model differs from the real FLRW metric in a few ways. First, it's missing two spatial dimensions, but that doesn't matter here. Second, the scale factor is wrong: a(t) ~ sin t isn't even a solution of the Friedmann equations, much less a solution approximating the world we see. Third, there's no reason to think that the real universe wraps around spatially like that. You can imagine "unrolling" the globe such that circling around it actually takes you to a new place (technically this is taking the universal cover of the sphere with the poles removed). Fourth, the early universe was opaque, so we should actually cut off the loxodrome some distance short of the south pole (i.e. when it hits Antarctica). But the basic geometry is right. This is why we can see things farther than ct away. -- BenRG (talk) 21:18, 17 March 2008 (UTC)[reply]
Hi. I'm not really an expert at this but is it possible that the light left the quasar, say, 12 billion years ago, and travelled to us now, so the image is really only 12 billion light-years away, but people calculated it based on the redshift of the quasar, and perhaps estimated that the quasar is currently 28 billion light-years away from us, as the expansion of space can travel faster than light in all those dimentions but matter itelf relative to itself can't? Thanks. ~AH1(TCU) 23:13, 17 March 2008 (UTC)[reply]
I'm not sure I understand your question. There is no image 12 billion light years away. As I said above (and below), the travel time times c is a completely meaningless distance. It has no physical significance at all. Light really does not travel 12 billion light years in 12 billion years. -- BenRG (talk) 23:57, 18 March 2008 (UTC)[reply]
Okay, I got inspired and made some numerically accurate images. These are two views of a surface that's just like the globe above except that the shape is calculated from the ΛCDM model and the WMAP five-year data. Click on either image for a larger version, and on the first one for some mathematical details of the embedding. The small circle at the "bottom" is 700 million years after the big bang. (Nothing earlier can be embedded with this particular choice of embedding parameters.) The large circle at the "top" is 18 billion years after the big bang, where you can start to see the accelerating expansion that eventually dominates in this model. The lines of latitude (purple) are 1 billion years apart. The lines of longitude (cyan) are one billion present-era light years apart. They're closer together than that in the past and farther apart in the future; in all eras the physical distance is proportional to the Euclidean distance measured along the lines of latitude. The cyan lines are also the worldlines of objects moving with the Hubble flow. The brown line is the worldline of the Milky Way and the yellow line is the worldline of the most distant known quasar with redshift z ~ 6.4 (I've assumed both are moving with the Hubble flow). The red line is the path of light from the quasar to present-day Earth, and the orange line shows the quasar's distance from Earth at the current cosmological time. You can verify by counting grid lines that the present-era distance to the quasar is about 28 billion light years and the light travel time is about 12 billion years. You should also be able to see that the light beam always makes a 45° angle with the nearby grid lines. (In the second image it kind of looks like it doesn't, but that's because of the perspective foreshortening.)
Every light beam currently reaching the earth follows the same path as the red beam, except possibly with an earlier or later starting time or from a different direction. So the region of spacetime we can see right now—that is, Earth's past light cone—is the subsurface traced out by the red line and its counterparts in other directions (most of which can't be drawn on this surface since two dimensions of space are missing).
Another interesting thing to notice is that the (real, metric) distance between the light beam and the Milky Way increases for the first few billion years before reaching a maximum and decreasing again. This is related to the odd fact that the angular diameter of extremely distant objects increases with distance—that is, the rules of perspective are reversed and more distant objects look larger. The cosmological time at which objects look smallest is the same as the time at which the light beam's distance is largest. The reason for this is left as an exercise for the reader (there's a simple geometrical answer).
I deliberately cut off the embedding short of a full circle to emphasize that space doesn't loop back on itself. The cutoff distance doesn't have any physical significance, and neither does the circumference of the full circle. It's an adjustable parameter of the embedding and I just picked a value that looked good.
All four of the colored lines are loxodromes (if that word is appropriate for a surface that's this much unlike a globe). The brown, red, and yellow lines are also spacetime geodesics, but the orange line isn't. N.B.: most geodesics are not loxodromes and most loxodromes are not geodesics. In fact they only coincide when v=0 or v=c, which happen to be the two cases that appear here. Also note that the spacetime geodesics (based on the real, Lorentzian geometry) are not the same as the Euclidean geodesics on this embedded surface, unfortunately. But I still think this kind of embedding is a useful way of understanding the expansion of the universe. It might be nice to have these images or similar ones in metric expansion of space. The expanding raisin bread model leaves something to be desired. -- BenRG (talk) 23:57, 18 March 2008 (UTC)[reply]
Holey-moley! Excellent detail, I'm bookmarking this for the future. I sure hope you're right, I'll be reading this for awhile :) Thanks for the effort! Franamax (talk) 00:17, 19 March 2008 (UTC)[reply]
Indeed! BenRG's analysis and explanation should definitely be preserved in an article. Also, kudos to Confusing Manifestation for helping the new user edit as well as providing responses along with 98... and Someguy. Truly a noteworthy thread. --hydnjo talk 20:36, 21 March 2008 (UTC)[reply]


March 17

Brushing teeth with bleach?

This question has been removed. Per the reference desk guidelines, the reference desk is not an appropriate place to request medical, legal or other professional advice, including any kind of medical diagnosis, prognosis, or treatment recommendations. For such advice, please see a qualified professional. If you don't believe this is such a request, please explain what you meant to ask, either here or on the Reference Desk's talk page.
This question has been removed. Per the reference desk guidelines, the reference desk is not an appropriate place to request medical, legal or other professional advice, including any kind of medical diagnosis or prognosis, or treatment recommendations. For such advice, please see a qualified professional. If you don't believe this is such a request, please explain what you meant to ask, either here or on the Reference Desk's talk page. --~~~~
--S.dedalus (talk) 02:39, 17 March 2008 (UTC)[reply]
I am not seeking medical advice. HYENASTE 02:48, 17 March 2008 (UTC)[reply]
Good because you're not going to get it. This is a stupid question. Theresa Knott | The otter sank 11:07, 17 March 2008 (UTC)[reply]
What does it say on the label on your bleach bottle? Read the part where it states the health risks. Why expect someone here to know more than the manufacturers who had lawyers work up the warning label? -- kainaw 02:38, 17 March 2008 (UTC)[reply]
It may be worth pointing out that the questioner seems to have been misled by the notion of "bleaching" teeth. When teeth are "bleached", it is done with specific preparations designed to whiten teeth, none of which contain household bleach (sodium hypochlorite). You don't bleach teeth with bleach. - Nunh-huh 14:21, 17 March 2008 (UTC)[reply]

what is the plant in this picture?

The large bushy plants after the grass in the foreground.

I was walking in the park with my boyfriend today and we could not identify the round bushy plants. They are in the background of this pic...Are they called pampa grass or somthing?W-i-k-i-l-o-v-e-r-1-7 (talk) 00:58, 17 March 2008 (UTC)[reply]

What part of the world is this in? -GTBacchus(talk) 03:23, 17 March 2008 (UTC)[reply]

Pacifica, California. I this Pampas Grass?W-i-k-i-l-o-v-e-r-1-7 (talk) 03:40, 17 March 2008 (UTC)[reply]

Hi, there's an illustration here[3] of purple needlegrass or it could be a kind of para grass in California. It looks like the lower green grass has grown through and up around the dieback. Julia Rossi (talk) 05:44, 17 March 2008 (UTC) (Sorry, not since it's blue-green and belongs to your main plant.)[reply]
I think you are right, that this is pampas grass (not a native plant, but common in that area). You are seeing this years fresh green growth coming up through the broken and tangled dry remains of last years leaves and stems. --169.230.94.28 (talk) 16:23, 17 March 2008 (UTC)[reply]

Did the Big Bang go bang

My understanding of the BB is that before it there was nothing. A split second after the BB was hydrogen and maybe some hellium. And that the "Big Bang" was an explosion. Like they said on the History chanel, "the Universe" exploded into existence. So I mentioned this to a friend and said that the Big Bang was an explosion. And that before that there was nothing. He said I was wrong on both accounts. I all but called him a morron. He is not an ID. He is saying that the BB was not an explosion. My Q: If it was not an explosion then what was it? Q: what was it that exploded? What was there before the BB? thanks cris —Preceding unsigned comment added by Loach (talkcontribs) 01:11, 17 March 2008 (UTC)[reply]

The Big Bang was an explosion, in the sense that it was a rapid expansion of energy and matter. The tricky bit about it is that it wasn't just a bunch of compacted stuff exploding into the surrounding space, it was an explosion of the space itself. And, at the same time that space came into being, so did time, so it doesn't really make sense to ask "what came before the Big Bang", since there was no before - a similar question in spatial co-ordinates would be "what's North of the North Pole?" Take a look at Big Bang and Timeline of the Big Bang for more details. Confusing Manifestation(Say hi!) 02:55, 17 March 2008 (UTC)[reply]


Thank you. That is more or less what I thought. Again, thank you for the help. —Preceding unsigned comment added by 68.209.69.103 (talk) 03:20, 17 March 2008 (UTC)[reply]

I could have been your friend. The terminology is very confusing. "Big bang cosmology" is a description of the evolution of the universe over the last 13.7 billion years. It's not a description of how the universe started. The model has a mathematical singularity called "the big bang singularity" or just "the big bang", which you can think of as a limiting state of infinite density, but nobody takes it seriously; it's seen as a sign that the model breaks down there. Among people who speculate about what should replace the singularity, the most popular models involve a universe with no particular beginning of time (that is, there's nothing inherent in the models that suggests a beginning of time, in contrast to the big bang model, which has a singularity at a finite time in the past). See also Age of the universe#Explanation. As to whether the expansion of the universe should be called an explosion, that's a matter of terminology, but I tend to think that it doesn't fit the dictionary definition of an explosion very well. For one thing, one normally thinks of an explosion as expanding outward into the surrounding space, and there's no space surrounding the universe. It's homogeneous. Note that if it was an explosion then the explosion is still going on; there's no transition point at which it stopped exploding and started just expanding. -- BenRG (talk) 12:05, 17 March 2008 (UTC)[reply]
A fun little bit of history -- the "Big Bang" was a term used first by Prof. Sir Fred Hoyle, one of the Big Bang's most outspoken critics. He had used the term disparagingly, and as a contrast to his "Steady State" model, but (assumedly to his chagrin) the phrase caught on first among the general public, and eventually in the scientific community. Bill Bryson's excellent book "A Short History of Nearly Everything" tells the tale entertainingly. Also see the obituary at [4].Vance.mcpherson (talk) 19:53, 17 March 2008 (UTC)[reply]

eggs

Do all eggs contain all of the amino acids, i.e., if forced to live in a survival situation with nothing but eggs how long could you survive with no other food? —Preceding unsigned comment added by 71.100.174.10 (talk) 01:55, 17 March 2008 (UTC)[reply]

Assuming you are referring to chicken eggs, they contain very little vitamin C. Without vitamin C, scurvy is very likely to occur. -- kainaw 02:36, 17 March 2008 (UTC)[reply]
Which is true, and pertinent to the second part of the question. As to the first part, yes, eggs contain all of the essential amino acids, which is the reason they are considered a "high quality" source of protein. But as Kainaw points out, the essential amino acids alone aren't enough to support health; there are other essential nutrients that are not amino acids. (Another problem is that eggs contain avidin, so a diet consisting entirely of eggs is likely to cause biotin deficiency. - Nunh-huh 03:02, 17 March 2008 (UTC)[reply]
As a followup question what basic items contain all of the nutients essential for indefinite survival? —Preceding unsigned comment added by 71.100.174.10 (talk) 04:56, 17 March 2008 (UTC)[reply]
I asked a question before about human milk, which, after all, is designed to provide for all human nutritional needs, at least for a baby. However, the nutritional label on regular cow's milk seems to indicate it is lacking in many nutrients we need. I forget if this discrepancy was ever explained. StuRat (talk) 05:57, 17 March 2008 (UTC)[reply]
(Not sarcasm here.) And that's why it's cow's milk and not us-milk, and is made up with supplements for human infants. There's a nice section about comparative milks here[5]. Julia Rossi (talk) 06:06, 17 March 2008 (UTC)[reply]
But human milk doesn't contain all the nutrients we need, like iron and folic acid: [6]. So how do babies survive on it ? StuRat (talk) 17:29, 17 March 2008 (UTC)[reply]
Because they're babies and have different nutritional needs to us. That's why, apart from the convenience factor, they have to be weaned as they grow up and start developing different nutritional needs. And why you can't feed a tiny baby puréed adult food instead of milk. Skittle (talk) 13:19, 18 March 2008 (UTC)[reply]
Babies do have different nutritional requirements, yes, like needing more fat and cholesterol to support rapid growth. However, they certainly must require iron for growth, as well. Don't babies have hemoglobin in their blood ? StuRat (talk) 14:30, 18 March 2008 (UTC)[reply]

It sounds like you saying that human milk and supplemented cow's milk provide complete nutrition for indefinite survival. What I am really looking for are combinations of other foods that one might search for and find in a survival situation like egg of all types for protein and pine needles for Vit C. I'm sure there must be lots of combinations so I am trying to find out what they are. Is there a chart somewhere perhaps that shows all of the required essentals with overlays of the portions provided by various foods? —Preceding unsigned comment added by 71.100.174.10 (talk) 07:43, 17 March 2008 (UTC)[reply]

Adventurers seem to know what food to take that doesn't weigh much and keeps them going for days (eg beef jerky, dried fruits, nuts and chocolate), but I don't know what that would be listed under. About milk, my take is that it isn't the complete food. I looked at space food but there's no information about nutrition just mechanics. Julia Rossi (talk) 08:53, 17 March 2008 (UTC)[reply]
Space food has some interesting information but I'm not looking for food which is pre-prepared but rather items which one might look for in the great outdoors, i.e., jungle, coastline, desert, etc. I'm not necessarily looking for foods you can take with you but rather foods that you can find. —Preceding unsigned comment added by 71.100.174.10 (talk) 15:35, 17 March 2008 (UTC)[reply]
You may be interested in some of the references of the Survival skills article. What food sources you might encounter in a wilderness would of course depend entirely on what biotope you are in – looking for mussels, for example, would be a great idea on the coast but less useful in an inland desert... So where exactly are you planning to get stranded? Also note that you can survive for weeks without any food at all, and for months or years on an incomplete diet. Are we talking about a permanent situation here, or a would crawling back towards inhabited areas be an option? --169.230.94.28 (talk) 16:13, 17 March 2008 (UTC)[reply]
In my youth when I ran away from home my inability to find a hamburger, French fries and a soda usually brought me to my senses and led me to the nearest pavement an on to the nearest fast food store. Now that I am older I'm curious just how long I can go out there somewhere nobody else is without need for finding pavement and inevitably French fried onion rings. —Preceding unsigned comment added by 71.100.171.236 (talk) 22:32, 17 March 2008 (UTC)[reply]

If you supplemented your eggs with fruits and vegetables I think you'd do pretty well. If you knew your edible plants better than how to steal eggs (you might decimate the local population rather fast depending on where you are eating only eggs), you could also do well combining grains with beans or nuts for your protein. Variety would be the key, some meat, some fruit, etc., since there's no one food you can healthily live on forever. — Laura Scudder 16:31, 17 March 2008 (UTC)[reply]

Sodium-potassium pumps

What percentage of the ATP that is used in the body is used by the brain to run the sodium potassium pumps? Schmultz (talk) 03:05, 17 March 2008 (UTC)schmultz[reply]

25%-30%, but don't reference me in your thesis. Mac Davis (talk) 04:46, 17 March 2008 (UTC)[reply]

Phosphodiester bonds

I want to know whether a molecule of water is removed when forming a phosphodiester bond between 2 DNA neucleotides. —Preceding unsigned comment added by 124.43.61.81 (talk) 06:42, 17 March 2008 (UTC)[reply]

Yes. Someguy1221 (talk) 09:24, 17 March 2008 (UTC)[reply]
See condensation reaction. Wisdom89 (T / C) 19:03, 17 March 2008 (UTC)[reply]

infrared raman spectroscopy

can infrared raman spectroscopy be used to detect bones in in vivo detection Neel shah556 (talk) 08:22, 17 March 2008 (UTC)[reply]

If the bones are covered by flesh, no. Infrared raman spec is used mostly on thin tissue samples or single cells. In these samples it can detect differences in the concentrations of proteins, lipids, carbohydrates and nucleic acids.--Shniken1 (talk) 11:11, 17 March 2008 (UTC)[reply]
Good answer ("that's using the old noodle"). :-) StuRat (talk) 14:48, 18 March 2008 (UTC)[reply]

Minimum entropy

Can whatever proceeded the Big Bang be defined as the state of minimum entropy?

There is no accepted physical theory on what preceded the big bang. In fact, accepted physics only deals with what happened after the very moment of the big bang. So there's nothing you can say about it. Someguy1221 (talk) 09:30, 17 March 2008 (UTC)[reply]
It is reasonable to say that some point in the early or very early universe there was a point of least entropy, and that entropy has increased since then, giving a direction to the cosmological arrow of time. The problem lies in trying to pin down just when this point of least entropy was. One big difficulty is that we have no agreed model for the physics of the very early universe (up to the end of the inflationary epoch) so we can't be sure that the second law of thermodynamics even applied then. One theory is that inflation itself drove entropy down, so that the point of least entropy is at the end of inflation, and the universe has been climbing out of this entropy "hole" ever since. Gandalf61 (talk) 12:15, 17 March 2008 (UTC)[reply]

swing??

why can't bernoulli's theorem be used to explain swing in cricket balls? --scoobydoo (talk) 14:19, 17 March 2008 (UTC)[reply]

because cricket is not a series of tubes? —Preceding unsigned comment added by 79.122.42.52 (talk) 14:59, 17 March 2008 (UTC)[reply]

I've read through the article and I suppose it might involve Bernoulli's Principle, but since I'm American half the words are complete Greek to me. The article doesn't really explain what swing does, can someone fix that? 206.252.74.48 (talk) 19:05, 17 March 2008 (UTC)[reply]
The Swing bowling article might help, but it is still full of cricket jargon. Perhaps the best explanation for american ears is to think of it like pitching a curveball, but using a different principle to get the ball to curve. Astronaut (talk) 03:36, 18 March 2008 (UTC)[reply]
Most ball sports players, can use Bernoulli's Theorem to cause the ball to curve in flight. I've seen it in football, golf, tennis, table tennis, baseball. However, I think Bernoulli's Theorem assumes the surface is of constant roughness. In cricket, bowlers will polish one side of the ball (by polishing it on their trousers), and occasionally cheat by roughening the other side (by stepping on the ball with their spiked shoes or scratching the surface with coins or dirt). Having one smooth side and one rough side affects the airflow around the ball and causes it to curve in flight - towards the rough side (I think :-)) where there is more aerodynamic drag. If the bowler has also put some spin on the ball, the overall effect can be very complicated. Astronaut (talk) 03:36, 18 March 2008 (UTC)[reply]
See also the related Magnus effect. -- Coneslayer (talk) 22:06, 19 March 2008 (UTC)[reply]

temp & fire

does temperature & fire have any relation? i mean, can i put out the fire of a burning body by cooling it?--scoobydoo (talk) 14:23, 17 March 2008 (UTC)[reply]

You might be able to - for instance if you manage to freeze it.
Also some things just won't burn below a certain temperature.87.102.13.144 (talk) 14:56, 17 March 2008 (UTC)[reply]

See our article on fire triangle. Fires do need heat. Rapidly cooling a fire (by adding lots of water for example) does put it out. Theresa Knott | The otter sank 15:25, 17 March 2008 (UTC)[reply]

NASA has a page here that shows how to snuff a candle by drawing the heat away from the wick with a coil of bare copper wire. --Milkbreath (talk) 16:47, 17 March 2008 (UTC)[reply]
There are basically three ways to extinguish a fire: deprive it of heat, fuel, or oxygen. Spraying water on a fire removes heat because the water absorbs heat when it is vaporized to form steam. Since water is inexpensive and readily available, this makes a good choice for most fires. It's not a good choice for some fires, like electrical fires (because water conducts electricity and could electrocute the firefighters) and oil fires (because water can splatter burning oil). In such cases they normally try to deprive the fire of oxygen by using halon or foam, or just wait until it burns itself out by using up the available fuel supply. StuRat (talk) 14:42, 18 March 2008 (UTC)[reply]
You can actually put out fires by causing an explosion nearby. The more you know... shoy 03:53, 20 March 2008 (UTC)[reply]

Speed of ...

Say you have a rigid object, such as a meter long titanium pole. When you pull one end, the other end presumably doesn't move instantly, because the force would have to move faster than the speed of light. It must be determined by the flexibility between the bonds of the titanium atoms/molecules. So ignoring the impracticality of its weight and size, if you were to pull a light-year, or some similar length, long titanium pole, the other end wouldn't notice it's been pulled until all the bonds are at their maximum pulling length? At that point, it still can't be instant if it's pulled further? Could anyone elaborate on what goes on? -- MacAddct  1984 (talk • contribs) 15:16, 17 March 2008 (UTC)[reply]

I'm not sure what you are asking but yes, it is impossible to pull all of it instantaniously. The rod will deform elastically and the pull will travel down it at the speed of sound in that material. (much slower than the speed of light). Theresa Knott | The otter sank 15:28, 17 March 2008 (UTC)[reply]
This is a Reference Desk Frequently-Asked Question but I think you've already sussed-out the answer: What we consider to be the "structural strength" of solid materials is actually the electromagnetic interaction of the electron shells of the constituent atoms. And these electromagnetic interactions can never propagate faster than the speed of light. So if you pull or push on that light-year-long titanium rod, a compression or expansion wave propagates through the material. It certainly doesn't go faster than the speed of light and probably only travels at the speed of sound in that material.
Atlant (talk) 15:31, 17 March 2008 (UTC)[reply]
Thanks! Yeah, that's about what I was thinking. It's hard to think in terms of familiar objects moving in ways we're not familiar with.
As for it being FAQ , is there an FAQ (official or unofficial) for the Reference Desk? If not, there really should be one started... -- MacAddct  1984 (talk • contribs) 15:53, 17 March 2008 (UTC)[reply]
There is a FAQ page, at Wikipedia:Reference_desk/FAQ. It is embryonic and underused, because it is hidden. We should link to it from the main RD page, and maybe mention it in the Before asking a question/Search first section at the top of each RD page. --169.230.94.28 (talk) 19:04, 17 March 2008 (UTC)[reply]
So what happens if you move one end faster than the speed of sound ? StuRat (talk) 17:36, 17 March 2008 (UTC)[reply]
You'd be breaking the rod. By definition you would be moving the atoms faster than they could convey that movement on to the atoms next to it; the rod wouldn't be structurally stable if you could do that, by definition. Remember it's the speed of sound in that material, not the speed of sound in air, which is normally what we think the speed of sound as being. --Captain Ref Desk (talk) 18:08, 17 March 2008 (UTC)[reply]
Or, if you are moving in a compressive direction, and buckling doesn't occur, you may create a shock wave. --169.230.94.28 (talk) 19:04, 17 March 2008 (UTC)[reply]
It's not speed that would break or buckle the rod, but acceleration 196.2.113.148 (talk) 22:07, 17 March 2008 (UTC)[reply]

I've been told that in the disease of Diabetes damage to the body is done by excessive amounts of either sugar or insulin molecules. Exactly what type of damage is done by excess amounts of either molecule besides oppressing the production/utilization of the other molecule that would cause the loss of toes? Is it damage to cell chemistry or structure, and if so of what does the damage consist? —Preceding unsigned comment added by 71.100.174.10 (talk) 15:47, 17 March 2008 (UTC)[reply]

Wikipedia has a very detailed article on diabetes, signs and symptoms, the different types, etc... -- MacAddct  1984 (talk • contribs) 15:56, 17 March 2008 (UTC)[reply]
If you're losing toes, you should go see the doctor. -- MacAddct  1984 (talk • contribs) 15:58, 17 March 2008 (UTC)[reply]
LOL... I'm hoping to learn what the sugar and the insulin do to the body that causes reports to be written of poeple with Diabeties to loose toes. Mine remain quit tasty, thanks, according to the wife. —Preceding unsigned comment added by 71.100.171.236 (talk) 21:40, 17 March 2008 (UTC)[reply]
Nothing in the post suggests that the poster is losing toes.
More specifically this is probably the section of the article you need. AlmostReadytoFly (talk) 16:52, 17 March 2008 (UTC)[reply]
this is much more what I'm looking for. Thanks.

It brings tears to my eyes...

...to waste part of an onion, but are the green sprouts in the center edible ? That is, should I make the best of them or make compost of them ? StuRat (talk) 18:07, 17 March 2008 (UTC)[reply]

They are edible, but you are only getting them as your onions are starting to grow. Try keeping your onions in a dark cool place, if in a hot country the fridge will do. This will stop the central green bit appearing. Taste wise the green bit will be slightly more tough than the cooked down flesh, more similar to a spring onion. I would not use it in a Burre Blanc sauce but it would be fine in a stir fry or stew. GameKeeper (talk) 19:23, 17 March 2008 (UTC)[reply]
Thx. StuRat (talk) 20:06, 17 March 2008 (UTC)[reply]
1138? 206.252.74.48 (talk) 16:01, 18 March 2008 (UTC)[reply]

1-Octen-3-ol CAS numbers?

I'm trying to track down the CAS numbers for 1-octen-3-ol. The best I could find is this page from a book about coffee. It lists the numbers [3391-86-4], (±) [50999-79-6], (R)-(-) [3687-48-7], (S)-(+) [24587-53-9]. The latter two clearly refer to the two enantiomers. My question is: what's the difference between 3391-86-4 and 50999-79-6? Both appear to refer to a mixture of the two enantiomers. Thanks, AxelBoldt (talk) 18:43, 17 March 2008 (UTC)[reply]

[3391-86-4] clearly states "matsutake alcohol FEMA 2805" almost certainly "Flavor and Extract Manufacturers Association".
The most probable explanation is that the natural product 'matsutake alcohol' was known before it was recognised to be indentical to 1-octen-3-ol, hence the different numbers.
The simplest way if possible would be to examine the papers referenced by [3391-86-4] and see exactly what they describe.87.102.13.144 (talk) 19:32, 17 March 2008 (UTC)[reply]
Also [3391-86-4] might refer to a naturally obtained product - if in the future this product is shown to contain other components then the distinction will be useful (for example conside oil of almond which is almost all benzaldhehyde - but not 100%)
Take a look at the papers cited for both from chemical abstracts - this might give you a clue as to what's going on eg does the use of one of the numbers cease after a certain date, or is the term 'matsutake' specific to certain countries ?87.102.13.144 (talk) 19:40, 17 March 2008 (UTC)[reply]
According to CAS, [50999-79-6] is a deleted CAS number and [3391-86-4] should be used. -- Ed (Edgar181) 19:44, 17 March 2008 (UTC)[reply]

Magical self cleaning wound

Yesterday I cut myself in my finger whilst outside. It had some dirt in it so I attempted to clean it to the best of my ability. However some small about of debris was still stuck inside. After a while I stopped trying to get the debris out, thinking it would most likely do more harm than good to go out of my way to pick out the last bit of dirt. Come next morning my wound was magically clean. And when I say clean I mean, perfect pink/reddish skin with not a hint of dirt. I'm wondering, where did this dirt go? Is there some mechanism by which wounds are able to expel small amounts of debris? Did it just fall out whilst I was sleeping even though I couldn't get it out myself? PvT (talk) 18:58, 17 March 2008 (UTC)[reply]

Blood would be the obvious agent to pour out of a wound and cleanse it. I've also noticed a clear fluid flow out of wounds not deep enough to bleed. I suspect that this is the blood plasma without the red blood cells, which need a larger opening to escape. Pus can also flow out, but only after the wound becomes infected. StuRat (talk) 19:13, 17 March 2008 (UTC)[reply]

Well the skin literally grows from inside to out - so it's unlikely to trap any dirt.. Once formed the new skin layer will be drier than a wound making it less sticky - and so any remaining dirt just fell off. Probably.87.102.13.144 (talk) 19:15, 17 March 2008 (UTC)[reply]

There is also the possibility that while you were asleep your unconcious mind went to work scraping the wound against the bed sheets or instructing other parts of your body to scratch or rub the wound. Sleep study video reveals all sorts of directed movements as if we were actually or part awake.

I wouldn't be so sure wounds are self-cleaning. When I was a child, I managed to stab myself in the hand with a newly sharpened pencil. Decades later, I still have a small grey dot under the skin. Astronaut (talk) 02:47, 18 March 2008 (UTC)[reply]
And I have a 20 year old splinter that hasn't managed to work it's way out, yet. StuRat (talk) 04:41, 18 March 2008 (UTC)[reply]
I too have a mark from a pencil stab (a "friend" did this to me in primary school). I think these are different from cuts as they are deep puncture wounds. Also, the graphite is organically neutral meaning that it won't be attacked by the immune system. I once had a thorn stuck in a deep cut, and it became an infected boil, and the thorn was eventually ejected along with puss. -- Q Chris (talk) 10:10, 18 March 2008 (UTC)[reply]
In my case I believe the wooden splinter eventually dissolved, leaving the dark brown wood stain which once coated the splinter behind as a type of tattoo. StuRat (talk) 14:19, 18 March 2008 (UTC)[reply]
As far as I know White blood cells do this trick, if that dosen't happen right away some white cells will die trying and form pus, which should do the trick then. As for some particles sometimes remaining in the body - I guess this is because new skin forms before immune system has managed to clean the wound, and even then sometimes stuff grown into the body is eventualy pushed out, but probably this happens to larger (or otherwise "more dangerous") stuff than dirt. ~~Xil...sist! 23:45, 20 March 2008 (UTC)[reply]

Searching for massively-multiplayer synonyms in PubMed

So "tight junction protein 1" has a variety of names; how am I supposed to search effectively in PubMed? Is there some way of referring to this gene/protein in such a way that all synonyms are searched for at once?

I might be wrong, but a little further down on that page it has all of the names in a list box ("Search for TJP1"). Hold down shift and you can select all of them at once. Change the boolean term to OR so that you get publications with any of those names in it (AND would get publications that contain each and every one of them). Hit search. Many results abound. --Captain Ref Desk (talk) 20:25, 17 March 2008 (UTC)[reply]
Have you ever worked with the MeSH function?
Change the drop down to "MeSH" and search for, say, tjp 1. The result shows zonula occludens-2 protein, and it the page lists the synonyms. On the same line of the substance name, all the way to the right, click on "Link", then on "Pubmed". Hopefully this will give you good results. 128.163.116.64 (talk) 22:18, 17 March 2008 (UTC)[reply]

Chem Acid Stoichimetry problem

Suppose I was given Ba(OH)2 at 0.5 M/L and was asked to find it's pOH. Since it's diprotic, I have to multiply the concentration by 2 right? If so, why do I multiply the concentration by 2? Why not the number of moles?


It will dissociate into Ba + 2OH right? 99.240.177.206 (talk) 21:04, 17 March 2008 (UTC)[reply]

Yes, Ba(OH)2 will dissociate into Ba2+ and two OH- (note charges). It's not really diprotic, as it has two hydroxyls, not two protons (diprotic refers to acids, not bases). The reason you multiply by two is that each mole of Ba(OH)2 contains two moles of OH-, so in a sense, 2 is the number of moles. BTW, this analysis only holds if the barium hydroxide is completely dissolved and completely dissociated. If it's a weak base it may stick around as dissolved Ba(OH)2 or possibly Ba(OH)+ (Sorry, I don't know the pKas of Ba(OH)2, so I can't give you a definitive answer.) Also, I don't think the solubilities work out - the barium hydroxide article lists the solubility as ~60 g/L (for the octahydrate), whereas a 0.5 M solution would be ~150 g/L (for the octahydrate). Lastly be aware that the capitol M already means moles per liter (molar). You don't have to divide through by liters again, unless you mean moles per liter per liter. -- 128.104.112.85 (talk) 21:46, 17 March 2008 (UTC)[reply]
If we're going to be pedantic about terminology, then it's the Kb, not the Ka.
Now, you have a molarity of .5M. Let's set up the first equation. Barium Hydroxide has a Kb of 104.71. Assuming that's for the reaction Ba(OH)2 --> Ba+2 + 2OH-, we have the following:
104.71=[Ba+2][OH-]^2/[BA(OH)2]
104.71=(x)(2x)^2/(.5-x) (where x is the number of moles/L disassociated)
104.71=4x^3/(.5-x)
x=.4954
We really want 2x, since we get 2 mol OH for every mol Ba(OH)2
2x=.9907
[OH-]=.9907
pOH=.00405
Odds are, this is completely wrong. --uǝʌǝsʎʇɹoɟʇs(st47) 21:55, 21 March 2008 (UTC)[reply]

Part of the USS Pampanito's hull

The USS Pampanito is the submarine pictured in this photo. Is there a term for the black part of its hull that tapers from the bow? Or is it still the hull? --BrokenSphereMsg me 21:08, 17 March 2008 (UTC)[reply]
If you mean the part that bulges around the waterline, I'd say tapers to. That said, it's certainly part of the hull -- or at least the light hull. On similar-era U-boats, those structures were "saddle tanks" -- auxiliary ballast tanks. Such tanks could also be designed to carry fuel, but the Germans left their diesel tanks inside the pressure hull to improve depth charge survivability. — Lomn 21:23, 17 March 2008 (UTC)[reply]
Whaddaya know? Saddle tank (submarine). — Lomn 21:27, 17 March 2008 (UTC)[reply]

Paraffin/aluminium foil

Years ago at a ski-lodge as after-dinner entertainment one of the guests carried out a trick involving a bit of chemistry, and I wonder whether anyone here knows how it was done. A piece of foil - the type found in some cigarette packs - had something done to it using paraffin/kerosene. The foil was then crumpled into a tight ball and placed in the palm of the hand. Within a few seconds it became too hot to hold. How was it done? —Preceding unsigned comment added by 196.2.113.148 (talk) 21:56, 17 March 2008 (UTC)[reply]

Probably caustic soda......Rotational (talk) 16:18, 18 March 2008 (UTC)[reply]


March 18

More quantum physics...

I need to know, in depth, all aspects of the behavior of electrons in crystaline structures (more specifically, quantum dots). If there is an article on this, please let me know. Thanks! Zrs 12 (talk) 03:01, 18 March 2008 (UTC)[reply]

Quantum dot seems like a good place to start. DMacks (talk) 04:07, 18 March 2008 (UTC)[reply]
Read it. I need to know in depth. I'm having to do a project over these things and apparently electrons act differently in them than in single atoms. (That probably sounds like I'm being short or rude, but I don't mean it like that). Zrs 12 (talk) 04:41, 18 March 2008 (UTC)[reply]
No problemo...wasn't clear from your question what if anything you already knew/had read:) By spacially constraining the electrons, the energy levels are changed, so the spectra of a quantum dot that are based on electronic transitions are different than for free atoms/molecules. Quantum well has some detail. What level and sort of project are we talking about here? "In depth" and "all aspects" sound either contradictory or unlimited in scope without knowing what kind of project and what academic level. The refs cited in quantum dot, quantum well, and potential well are good ones. DMacks (talk) 05:12, 18 March 2008 (UTC)[reply]
Thanks for your help. Well, I am in 9th grade. However, I have been reading alot about the subject and am beginning to grasp some of the more in depth concepts such as the mathematical concepts in some areas of quantum physics. This is also a project to be in the state science fair, so it probably must be done at a higher that 9th grade level to have any chance. By the way, the specific information I seek about the electrons in quantum dots is; Why are the electrons confined to bands rather than descrete energy levels where they can have partial quanta of energy (right?). And, how do the electrons behave. They bounce back and forth, but between what? Where? Are they confined to a specific area? How and why are the energy levels changed by spatial restriction? To how big an area does the Pauli exculsion principle apply? To a single atom? A molecule? What do the matrices of objects such as represent?Bra-ket notation Is it correct to say that, for the photoelectric effect to take place in which Eg is the energy of the bandgap, h is Planck's constant, and f is the frequency of the light?
Once again, thanks very much for your help. Zrs 12 (talk) 05:45, 18 March 2008 (UTC)[reply]
As the Pauli Exclusion Principle article states, it says that no two fermions may occupy the same quantum state simultaneously, fermions are a type of elementary particle, described by The Standard Model. I recommend purchasing the book Hacking Matter at the local bookstore, about quantum dot, wells, and artificial atoms. I suggest taking up your discussion at Physicsforums's physics section— they will be very helpful to you.[7] Mac Davis (talk) 10:49, 19 March 2008 (UTC)[reply]

Infrared raman spectroscopy

Infrared rays having best penetration in skin and flesh, why can't be utilized for in-vivo non-invasive probing? Can't these rays yield good signature of bones? Neel shah556 (talk) 04:00, 18 March 2008 (UTC)[reply]

Infrared is strongly absorbed by many of the components in skin and flesh. Water, lipids, proteins etc. Infrared therefore does not have good penetration of skin and flesh. X-rays on the other hand are not absorbed (much) by flesh and therefore penetrate the skin and flesh to the bones, thus x-rays are used to look at bones in-vivo, not infrared.--Shniken1 (talk) 12:40, 18 March 2008 (UTC)[reply]

global warming

Will rise in sea level be even across the world? Or will the rise be more in regions near to arctic and antarctic and less in regions near equator? —Preceding unsigned comment added by 59.92.113.83 (talk) 06:46, 18 March 2008 (UTC)[reply]

Sea level changes are not uniform, but it's not as simple as greater rises at higher latitudes; contributing factors are tides, density due to salinity, expansion and contraction due to heat, flows around landmasses, etcetera. Most models show that the greatest rise will be seen in the arctic (due to increased runoffs causing a drop in salinity) and the lowest rise will be seen in the antarctic. FiggyBee (talk) 07:34, 18 March 2008 (UTC)[reply]
Dumb question then - why wouldn't the higher sea level water flow down to the lower level until the levels are equal? Franamax (talk) 08:12, 18 March 2008 (UTC)[reply]
It does, but not fast enough to overcome all the other factors (which are all ongoing processes, rather than one-off starting conditions). FiggyBee (talk) 08:19, 18 March 2008 (UTC)[reply]
Thanks! I knew it was a dumb question. :) Flow varies with square-root of water head, right? It's just hard to imagine that the Arctic Ocean could fill up with water faster than it could empty out. I forgot the factor of time. Franamax (talk) 08:39, 18 March 2008 (UTC)[reply]
I agree with you Franamax. While some difference seems inevitable I have a hard time imagining a significant difference. Are we talking about fractions of an inch here ? StuRat (talk) 14:24, 18 March 2008 (UTC)[reply]
From Sea level : "Mean sea level does not remain constant over the surface of the entire earth. For instance, mean sea level at the Pacific end of the Panama Canal stands 20 cm higher than at the Atlantic end." 72.10.110.107 (talk) 15:22, 18 March 2008 (UTC)[reply]
The earth's spin alone accounts for the height of the Pacific being slightly elevated relative to the Atlantic, as I understand it, and that difference is more pronounced towards the Pacific's West end. Any of us who have spent time on bodies as small as the Great Lakes know that water level can seem to rise or fall significantly due to changes in wind direction. And, for that matter, the Great Lakes do have a flow -- a molecule of water at the West end of Lake Superior takes 15,000 years to reach the Atlantic. So water, as it were, doesn't settle and "even out" quite so quickly as we might suppose.Vance.mcpherson (talk) 15:41, 18 March 2008 (UTC)[reply]
But, of course, the Great Lakes are far more restricted in their flow to the ocean than the oceans are to each other. If the oceans were only connected via narrow channels, it would be easier to understand a difference in elevations. The tides will also cause some areas to have far higher elevations than others at any given time. However, none of this explains why the relative elevations would significantly change when more water is added to the system via global warming. StuRat (talk) 22:15, 18 March 2008 (UTC)[reply]
The Arctic Ocean may be the exception though, it only has two outlets, and some pretty big rivers flowing in, Mackenzie, Ob-Irtysh, Yenisey (I think). Plus thermal expansion as the ocean warms up, so it's plausible. I would be interested to see the models though. Rising sea levels are certainly already a problem in the Arctic. Most interesting... Franamax (talk) 01:28, 19 March 2008 (UTC)[reply]
A rise in sea level is not necessarily even around the world. As discussed above, mean sea level (MSL) involves complex measurement and it is attempted to accurately determine an MSL using the geoid as a level reference surface. A rise in sea level, however, would expand the geoid and mean sea level. Isostatic changes will also do this. Mac Davis (talk) 10:42, 19 March 2008 (UTC)[reply]
Hi. Well, I read in a book that a recent satellite measurement of the world's oceans that the mean sea level excluding waves and other anomalies in a given area was actually greatly affected by the sea bottom elevation: the surface water was generally lower near a trench and higher near an undersea mountain range. Also, a shift in sea currents could affect the amount of heat in a given area and thus the sea level. Erosion will also cause the now-undersea landforms to erode, allowing sea levels farther inland. Given the right (or should I say wrong) type of rock, the sea can either flow into the groundwater and flood a nearby low basin or create waves that enter the basin. For now, many regions in the arctic are still experiencing the shoreline recede, as the rising of the land since the last ice age is currently faster than the sea level rise, at least until the Greenland and Antarctic ice sheets start collapsing. Also, if you try to use the flood maps at flood.firetree.net, they are largely innacurate (see their disclaimer). Constant flooding in an area may also allow sea levels to creep up. If there is a river, the sea can easily flood farther inland bacause of the eroded lowlands formed by the river. Hope this helps. Thanks. ~AH1(TCU) 21:11, 19 March 2008 (UTC)[reply]


  I belive that the sea level will not be even around the world but I do not know the answer to your second question.  But i promise I will try to find out.
                                SilverLeaf209.226.138.43 (talk) 18:22, 23 March 2008 (UTC)[reply]

MSM expiry

This question has been removed. Per the reference desk guidelines, the reference desk is not an appropriate place to request medical, legal or other professional advice, including any kind of medical diagnosis, prognosis, or treatment recommendations. For such advice, please see a qualified professional. If you don't believe this is such a request, please explain what you meant to ask, either here or on the Reference Desk's talk page.
This question has been removed. Per the reference desk guidelines, the reference desk is not an appropriate place to request medical, legal or other professional advice, including any kind of medical diagnosis or prognosis, or treatment recommendations. For such advice, please see a qualified professional. If you don't believe this is such a request, please explain what you meant to ask, either here or on the Reference Desk's talk page. --~~~~
TenOfAllTrades(talk) 12:55, 18 March 2008 (UTC)[reply]
You should ask a pharmacist and your doctor about this. They know much more than we do. Franamax (talk) 08:00, 18 March 2008 (UTC)[reply]

Doctors and pharmacists are Wikipedia editors as well.

This is not about pharmacy or medicine. The question is: Is MSM oxidized or reduced in household storage (or does it decompose)? [household chemistry]

MSM is not pharmacy; It is a mineral (sulfur) nutrient, much like calcium or magnesium. But the question is not so much about nutrition as it is about msm decomposition in household storage. Please leave this for discussion. —Preceding unsigned comment added by Zaqry (talkcontribs) 19:42, 18 March 2008 (UTC)[reply]

Is MSM oxidized or reduced in household storage (or does it decompose)? —Preceding unsigned comment added by Zaqry (talkcontribs) 22:39, 18 March 2008 (UTC)[reply]

Does MSM decompose in household storage and how (oxidation or reduction reaction for example)? —Preceding unsigned comment added by Zaqry (talkcontribs) 18:00, 19 March 2008 (UTC)[reply]

Radio transmitter power

A 50,000-watt radio station is transmitting at full power.

a) Is there at least 50KVA (50KVA plus the heat coming out of the building) running through the power lines into the station at all times?
b) Does it matter whether anyone is talking over the radio?
c) Is the answer different whether it's AM or FM, or VHF or UHF television?

Thanks for the help! Franamax (talk) 08:08, 18 March 2008 (UTC)[reply]

My understanding is

a) Yes for FM. See c
b) No for FM, but for AM it will increase and decrease as someone talks. There is a variation of AM known as Single-sideband modulation (SSB) that only uses power when there is some sound.
c) For FM and AM yes. For TV, I am not sure but I think that US digital TV uses ATSC Standards a type of SSB. I would imagine that in video there are no "silent" pauses like in audio, so the power load will be more constant. European digital TV uses Orthogonal frequency-division multiplexing, which I am unsure about. Analogue TV uses FM, and will therefore have constant power. -- Q Chris (talk) 10:02, 18 March 2008 (UTC)[reply]
For frequency modulation, phase modulation, and ordinary analog amplitude modulation (of 100% or less modulation depth), the average power of the carrier wave remains constant so the power inflow to the building will always be at least 50 KW. For single-sideband modulation, there's no transmitted power when no one is talking so the average power of the carrier wave varies with the average volume of the modulating signal. Analog TV uses vestigial sideband modulation (a hybrid of AM and SSB) for the video and frequency modulation for the audio.
Atlant (talk) 12:04, 19 March 2008 (UTC)[reply]

Atomic Radio clocks in Bulgaria

I live in Bulgaria. How do I find out whether an atomic radio clock will work in Bulgaria? Do I have to obtain one from a specific provider? —Preceding unsigned comment added by ENiklaus (talkcontribs) 12:06, 18 March 2008 (UTC)[reply]

Bulgaria seems to be at the edge of the region where the German DCF77 signal can be received. According to the PTB, the range is 2000 km (unfortunately that's probably not very helpful, but maybe interesting). Icek (talk) 12:54, 18 March 2008 (UTC)[reply]
(edit conflict)
There are long-wave time transmissions from Switzerland and Germany which might reach Bulgaria (depending on mountains etc.) They broadcast on slightly different frequencies, so, yes, you do need to purchase the correct receiver, and there is no guarantee that it will work where you live. I have found that the UK transmission can be picked up in some parts of buildings, but not in other parts. Mountains and steel frames cause reception problems. See Radio clock for more details, but the price of radio clocks is now quite low, so if they are not available for sale in your area, then it is probably because there is not a good radio signal. A world-wide time transmission can be picked up by a GPS clock (built into most GPS units) but you need to be either outdoors or near to a large window to pick up the satellite signals. dbfirs 13:08, 18 March 2008 (UTC)[reply]

Genetic probabilities

In a problem I'm trying to solve, there are 2 alleles, A and a. a is a defective allele, so the genotype aa induces disease. Both parents are carriers: Aa.
1) What is the probability that all three children are of normal phenotype? The way I see it, the chance of the normal phenotype in one child is 3/4, the result of a simple Aa x Aa Punnett square, so (3/4)3 = 27/64.
2) What is the probability that one or more of the children have the disease? Shouldn't it be 1/4 + 1/4 + 1/4 = 3/4? But 27/64 + 3/4 ≠ 1 ... help! Thanks, anon. —Preceding unsigned comment added by 70.23.85.120 (talk) 16:48, 18 March 2008 (UTC)[reply]

You can only add probabilities when they are probabilities of mutually exclusive (i.e. incompatible) events. Child A has the disease with probability 1/4, same for B, but the probability that A or B or both have the disease is not 1/4 + 1/4 because you are double-counting the case where both A and B have it, which has probability 1/16. So the probability for two children would be 7/16, which equals 1 - 9/16 that you could compute from the probability that both were normal. With three children it is easier to start with the probability that all three are normal, and you have that already. --Anonymous, 16:58 UTC, March 18, 2008.
As an exercise to help understand the probabilities you might want to list every possible combo. Each child has 4 possible combos (AA, Aa, aA, or aa). While Aa and aA both mean they are carriers of this recessive disorder, it's probably easier to list them separately as equal probability than combine them together. So, that gives you 4^3 or 64 possible combos. List them out and record how many children have the disease or are carriers for each combo. StuRat (talk) 22:05, 18 March 2008 (UTC)[reply]
Thanks so much :-), anon. —Preceding unsigned comment added by 141.155.38.85 (talk) 17:17, 19 March 2008 (UTC)[reply]

300,000 Elephants in Chad?

In the article excepted below [and in many secondary & tertiary sources] the assertion is made that “--the Chad population was over 300,000 animals as recently as 1970 and has been reduced to approximately 10,000 as of 2006.” I have looked carefully at the publications of the African Elephant Specialist Group of the IUCN for confirmation of these figures [latest report 2007]. The IUCN folks report no data on Chad elephants before 1985. They report “no evidence” for 1981 and no figures before that time. Where did this 300,000 figure originate and who started it?

2006 Zakouma elephant slaughter refers to a series of poaching massacres of African elephants in the vicinity of Zakouma National Park in southeastern Chad. These killings have been documented in aerial surveys conducted from May through August 2006 and total at least 100 animals.[1] This region has a four decade history of illegal killing of this species; in fact, the Chad population was over 300,000 animals as recently as 1970 and has been reduced to approximately 10,000 as of 2006. The African elephant nominally has Chadian governmental protection, but the implementation practices of the government (backed with certain EU help) have been insufficient to stem the slaughter by poachers.[1] The species African Bush Elephant (Loxodonta africana) occurs in several countries of Eastern Africa. —Preceding unsigned comment added by Lynmil (talkcontribs) 19:45, 18 March 2008 (UTC)[reply]
The first reference for 2006 Zakouma elephant slaughter is where that figure comes from, but trying to track that number further back was not very successful. It is amazing how many sites quote that "300,000 in 1970" figure and how frustrating it was to try tracking the source. I was unable to find anything approaching an original source, but I think I may have a good idea on how that figure was calculated. I think it may have been a backward calculation based on the Manovo-Gounda St. Floris National Park (Central African Republic) (N 475) report or some report like it. In that report, which is not specific to Chad but could reasonably be extended in principle to all surrounding areas, they report that the region had lost 95% of its elephants. It wasn't really clear to me what time frame they were referencing. Ninety-five percent loss since when? They were probably calculating the loss since poaching became a serious problem (starting around 1970 ???). Using that figure, you simply multiply the elephant population, at the time of the report, by twenty to find the starting population. Of course, to know that an area had suffered a 95% loss means UNESCO already had to know the starting population by some other means. Althought I don't doubt the figure at all, I would like to see the original source for that 300,000 quote. SWAdair | Talk 10:26, 22 March 2008 (UTC)[reply]

The IUCN report titled African Elephants and Rhinos: Status Survey and Conservation Report (1990) gave a 1987 estimate of between 200-400,000 elephants for Central Africa including Cameroon, Central Afr. Rep, Chad, Congo, Equat. Guinea, Gabon and Zaire [Now Dem Rep. of Congo]. Nearly half (195,000) of these animals were estimated for Zaire.

The IUCN 2007 African Elephant Status Report: An update from the African Elephant Database gives the Probable number of elephants in Central Africa as 48,936, with the Probable number from Dem Rep of Congo as only 7,955 animals. Thus Zaire (Dem Rep Congo) HAS lost 95% of the estimated elephant population from 1987 to 2007. The upper estimate for Central Africa in 1990 was 375,800. The upper estimate for 2007 is 48,936; thus an estimated loss of about 76%.

I agree with “SWAdair” that someone probably extrapolated from Central Africa data to Chad, but the loss of almost 300,000 savanna elephants from Chad could not have gone unnoticed. The figure is fictitious. Lynmil (talk) 19:38, 24 March 2008 (UTC)[reply]

Paramagnetic metal that do not become magnetized.

The question is what are metals called that are paramagnetic,but do not become magnetized (they are attracted by magnets, but when removed from magnetic infuences, are not magnetic)?

What metals are these? How do I reference the subject on WikiPedia?

Thank You, My Email address (removed for your protection)—Preceding unsigned comment added by 151.202.62.236 (talk) 19:55, 18 March 2008 (UTC)[reply]

"soft magnets", see magnet also remanence and follow the links in the 'see also' section.
The remanence of a material is the extent to which the material retains the magnetic field - you are obviously asking for 'low remanence'. Please ask for furhter info.87.102.47.176 (talk) 20:15, 18 March 2008 (UTC)[reply]
You may want to have a look at [[8]] for a little fun. I saw a nifty powerpoint by a fellow from University of Victoria on paramagnetism some six years ago, and I'll be damned if I know how to find him. If memory serves, and the link I provide confirms, paramagnetism isn't so much about what metal is paramagnetic as what you did to the metal, i.e. what else is it bonded to. Gets into the realm of organic chemistry.Vance.mcpherson (talk) 20:15, 19 March 2008 (UTC)[reply]

What is cyclooctatetraene?

What is cyclooctatetraene? 66.81.43.129 (talk) 21:59, 18 March 2008 (UTC)[reply]

Take a look at our article, Cyclooctatetraene. It appears to have a lot of info. -- Flyguy649 talk 22:02, 18 March 2008 (UTC)[reply]

March 19

Insanity...

Is the capability to display behavior patterns indicative of insanity in any way proof that the insane entity had a mind of its own to lose in the first place?

Just musing to myself about those neglected, stir-crazy, obsessive-compulsive, self-mutilating parrots at this late hour... --Kurt Shaped Box (talk) 02:42, 19 March 2008 (UTC)[reply]

While it certainly is possible for any creature with a brain to suffer a brain malfunction, I'd expect that complex minds are more likely to have complex malfunctions. So, strange obsessive-compulsive behavior may indeed be a sign of a fairly complex mind. A simpler mind would likely suffer from a more basic malfunction, like walking in circles instead of straight. I suggest you get the parrot in question a companion (or at least a cuttlebone to cuddle with). :-) StuRat (talk) 05:45, 19 March 2008 (UTC)[reply]

Biology: What are these caterpillars doing?

What are the caterpillars doing in this video? - Pureblade | Θ 04:46, 19 March 2008 (UTC)[reply]

I would speculate that they are exhibiting a defensive behaviour by co-ordinated action to give the impression of a larger creature. They may also be releasing body hairs containing a very powerful irritant. It is entirely possible that they sense the presence of the video operator and are reacting appropriately. I have experienced a similar effect with pine processional moths in Spain, when I blew on them gently they all jerked simultaneously. This seems to me to be an analogous behaviour. What is slightly odd, if these are pine processional moths, is that they are out feeding in the daylight. Of course they could be a related species demonstrating similar behaviour. Richard Avery (talk) 08:32, 19 March 2008 (UTC)[reply]
To me it looks like a sped-up version of the Macarena. Clearly somebody should tell these little buggers that it isn't cool anymore, if it every was. --Captain Ref Desk (talk) 19:49, 20 March 2008 (UTC)[reply]
But more honestly: I think they are Sawfly larvae. As to how they know to do this, this article is rather evocative. --Captain Ref Desk (talk) 19:49, 20 March 2008 (UTC)[reply]

Science in my gloves

We have to wear latex gloves in the cleanroom that I work in. Under these gloves we can wear a nylon glove liner. It's just a loose fitting liner so the glove isn't directly on your skin. I've noticed that sometimes as the night goes on, stains will develop inside my glove on the liner. The stains are blue like a bit of ink has found its way into the glove but the liners are white and stitched with white thread. There isn't any obvious source for the blue color. Some of the liners can be yellowed, almost as if they have been sitting in the sun, so I tried one of those to see if maybe the blue was due to a reaction between my sweat and whatever was yellowing the liners. I've only tested this once so far but the yellowed liner produced the blue stains while a whiter liner on my other hand did not. So what could be producing the blue color? Dismas|(talk) 08:16, 19 March 2008 (UTC)[reply]

an off-the-wall guess.... povidone/iodine (from gloves or handwashing) + starch --> blue stain. - Nunh-huh 08:53, 19 March 2008 (UTC)[reply]
I guess I should have mentioned this but we are required to wash our hands using this white foamy soap before putting the gloves on. Dismas|(talk) 09:01, 19 March 2008 (UTC)[reply]
Hi. This may be unrelated to your question, but usually in winter sometimes I wear normal gloves on the inside and waterproof ones on the outside. If the outside of my gloves get wet, sometimes the inside will too, and I often find disgusting colour marks on my hands afterwards. Talk about a green thumb, eh? Hope this helps. Thanks. ~AH1(TCU) 21:20, 19 March 2008 (UTC)[reply]

Two questions

What proportion of the scientific community believes the earth to be less than 100,000 years old and approximatly how accurate is carbon dating?--193.120.116.177 (talk) 11:02, 19 March 2008 (UTC)[reply]

And how old does the pope beleive the earth to be?--193.120.116.177 (talk) 11:18, 19 March 2008 (UTC)[reply]
Age of the Earth might help.
Zain Ebrahim (talk) 11:25, 19 March 2008 (UTC)[reply]
The Roman Catholic church does not have any theological issues with the earth being 4-6 billion years old. See Evolution and the Roman Catholic Church for related information. Specifically note what Cardinal Ratzinger (now Pope Benedict) said in a report: "According to the widely accepted scientific account, the universe erupted 15 billion years ago in an explosion called the 'Big Bang' and has been expanding and cooling ever since. Later there gradually emerged the conditions necessary for the formation of atoms, still later the condensation of galaxies and stars, and about 10 billion years later the formation of planets. In our own solar system and on earth (formed about 4.5 billion years ago), the conditions have been favorable to the emergence of life." -- 14:38, 19 March 2008 (UTC) —Preceding unsigned comment added by 128.104.112.85 (talk)
Do you have a reference for this?Zain Ebrahim (talk) 14:43, 19 March 2008 (UTC)[reply]
Found one: [9]. Zain Ebrahim (talk) 14:47, 19 March 2008 (UTC)[reply]
Carbon_dating#Calibration regarding the accuracy of carbon dating and radiometric_dating#Limitation_of_techniques for accuracy in dating the age of the earth. Lanfear's Bane | t 11:37, 19 March 2008 (UTC)[reply]
Carbon dating is just one kind of radiometric dating. It isn't used for determining the age of the Earth—it is only useful for dating things in smaller time periods than that. Uranium-lead dating is more common for things as old as the Earth.
As for how much of the scientific community believing the earth is less than 100,000 years old, practically none—a tiny number, insignificant when compared to the number of all scientists, and of those small number I imagine even smaller of them are active participants in a field of scientific inquiry that actually informs such assessments, or in what are considered to be real scientific institutions (not the Institute of Creation Research, for example).
There are quite a number of people outside the scientific community who believe that, but there is simply no compelling scientific evidence that the Earth is so "young" and much compelling evidence against the idea. I don't think it's an overstatement to say that the only people who believe the Earth is less than 100,000 years old are people who have a predisposition towards wanting to believe that on account of their religious beliefs.
As for the Pope, I am not aware of the current Pope's point of view vis a vis the age of the Earth. However perhaps you are referencing the work of the long-since-deceased Archbishop Ussher, who calculated that the Earth was only around 2,000 years old. --Captain Ref Desk (talk) 19:32, 19 March 2008 (UTC)[reply]
To be slightly more accurate than 'insignificant', Institute for Creation Research claims around 500 scientists (by which I think they mean those with a PhD in a scientific subject) that subscribe to their views. I'm not sure how many scientists there are in the US, but my guess is that this is substantially less than 1%. Interestingly it probably means that only a tiny fraction of scientists who are also Christians believe that the earth is less than 100,000 years old. DJ Clayworth (talk) 17:46, 19 March 2008 (UTC)[reply]
And it's the sort of number that gets smaller when you take their list and weed out all of those who 1. have degrees from non-accredited universities, and 2. are actively publishing or doing research in subjects related to things like the age of the Earth. I'm betting you'd cut the number down by quite a lot if you just took out the number of engineers—no offense made to engineers, but they seem to always predominate in the "people who think they really understand science deeply but actually have a fairly superficial working knowledge, and as such often overestimate their own competency in subjects they have not really studied seriously" category. --Captain Ref Desk (talk) 19:32, 19 March 2008 (UTC)[reply]
The precise stand of the Vatican on the Big Bang, Creation vs. Evolution, etc., can be found here: [[10]], at the Holy See's website; scroll down to paragraph 62. Other Christian denominations are not nearly so centralized, but in general the Anglican Communion has distanced itself from the notion of a 10,000 year old earth. In fact, in the West, the remaining "literalists" are largely Protestant denominations, many of whom have no fantastic claim to antiquity themselves. Vance.mcpherson (talk) 20:38, 19 March 2008 (UTC)[reply]

How do you poison a dress?

(I'm not sure whether I should post this at the Humanities-desk or here, but I figure since it has to do with medicine than history, Science would be most appropriate)

I saw the movie Elizabeth (the one with Cate Blanchett) a while ago, and something struck me: there is a subplot in it about an attempted assassination of Elizabeth using a dress that had been poisoned, so that when she would put it on, she would die (the attempt failed, obviously). That seems to me to be a not uncommon motif in literature, poisoning a dress. The most obvious other example is Medea killing Creon and Glauce by poisoning a dress.

Is this possible? I mean, can you make a dress that otherwise looks (and smells) normal, but when you touch it or wear it, it will poison you? Supposing you could, could you do it in the 16th century? I mean, if you just dunk it in a vat of arsenic-contaminated water (or something), wouldn't it be all discolored and also smell pretty foul? And is this just part of mythology, or has there ever been an attempt of using this method of assassination? --Oskar 14:45, 19 March 2008 (UTC)[reply]

It would be possible, but it seems to be firmly in the territory of soap opera and mythology. - Nunh-huh 15:43, 19 March 2008 (UTC)[reply]
So say you were a 16th century cardinal who didn't like all this Church of England nonsense, how would you do it? --Oskar 17:38, 19 March 2008 (UTC)[reply]
Get out of town? - Nunh-huh 17:49, 19 March 2008 (UTC)[reply]
Maybe it was soaked in cyanide with a nice almond scent. -- MacAddct  1984 (talk • contribs) 16:14, 19 March 2008 (UTC)[reply]
Well, the CIA considered giving Fidel Castro a poisoned wet suit, so why not? --Sean 17:11, 19 March 2008 (UTC)[reply]
Coupla drops of dimethyl mercury would do it (but long-drawn-out, not quick-convulse&die), but not sure anyone in the 1500s would be able to handle that stuff. Heck, even today it is hard to handle safely. DMacks (talk) 19:14, 19 March 2008 (UTC)[reply]
Pyrrole might be a good candidate, as it is a permeator. See [[11]]. Have no idea how they'd get the stuff in Elizabethan times, though, the synthesis of it is complex ([[12]]).Vance.mcpherson (talk) 20:50, 19 March 2008 (UTC)[reply]
Lots of things permeate skin, but need something that is reliably toxic in a dose small enough to be delivered that way. The MSDS you cite lists an acute LD50 of 98 mg/kg (mouse, orally), so even if that's the right ballpark for (human, dermal), we're talking several dozen mL to absorb. In smaller doses, it's not that toxic (see note on pyrrole about its use in cigarettes). DMacks (talk) 21:00, 19 March 2008 (UTC)[reply]
I like the idea of having small poison-tipped spikes somewhere inside the dress. Mac Davis (talk) 01:47, 20 March 2008 (UTC)[reply]
How about soaking the dress in nicotine ? It's absorbed through the skin and can cause death if enough is absorbed transdermally. Tobacco farmers must wear gloves when harvesting tobacco leaves, especially after a dew or rain, as concentrated nicotine in the rain drops will pass through the skin on the hands and can cause death. StuRat (talk) 04:17, 20 March 2008 (UTC)[reply]
Of course it works. Just ask Hercules. shoy 16:42, 20 March 2008 (UTC)[reply]
I think the trick is social engineering. Even if the soaking is visible and can be smelt, you could convince the target that it's a "refreshing" or "wellness" shirt or something. – b_jonas 17:21, 20 March 2008 (UTC)[reply]

hey

wats is the common and scientific name for sponges,flat worms, earthworms, bugs, frogs, jellyfish, round worms, clams, and starfish —Preceding unsigned comment added by 165.29.165.253 (talk) 15:14, 19 March 2008 (UTC)[reply]

Well, you just listed their common names. Scientific names will depend of each species of the animal you want; out of those groups there are probably millions(?) of different species. The only group they all fall under is the kingdom "Animalia". -- MacAddct  1984 (talk • contribs) 15:27, 19 March 2008 (UTC)[reply]
Simple enough eg type 'sponge' into the search box gives the sponge page which tells that they are all of the phylum 'Porifera'. You can do the same for the rest. type 'insects' for 'bugs'.83.100.183.180 (talk) 15:50, 19 March 2008 (UTC)[reply]

Well the whole collection together could be animals! or cold blooded animals Graeme Bartlett (talk) 05:13, 20 March 2008 (UTC)[reply]

Invertebrates? Sandman30s (talk) 07:55, 20 March 2008 (UTC)[reply]
...apart from the frog, of course. Gandalf61 (talk) 10:50, 20 March 2008 (UTC)[reply]
And invertebrates is a fairly silly name based on exclusion rather than inclusion, i.e., defined as animals without a backbone. Proper nomenclature is never based on exclusion it's just a vertebrate (human) bias. --jjron (talk) 14:10, 21 March 2008 (UTC)[reply]

Tantalum hafnium carbide

We don't have any reference in the article Tantalum hafnium carbide, the substance with the highest melting point known (?). Where is the publication on the 4488 K melting point? Thanks in advance. Icek (talk) 15:55, 19 March 2008 (UTC)[reply]

This Britannica article is a very reliable source. --Bowlhover (talk) 16:00, 19 March 2008 (UTC)[reply]
Thank you, but does anyone know where to find the original publication? Icek (talk) 16:12, 19 March 2008 (UTC)[reply]
Google told me that the phrase "The alloy tantalum hafnium carbide. (Ta. 4. HfC. 5. ), with a melting point of 42158C,. is one of the most refractory substances" appears in "11 Hafnium" of doi:10.1002/9783527619634.ch32, but I don't have access to the actual text from here. A publication like that is probably well-footnoted to primary and/or reputable secondary sources. DMacks (talk) 19:12, 19 March 2008 (UTC)[reply]

New car smell

I just found out today that the new car smell is toxic and carcinogenic. So what should we do to avoid it? Would opening the windows help? --Lazar Taxon (talk) 17:26, 19 March 2008 (UTC)[reply]

Do you have a reference for this? It would surprise me (at least, in the US, with all our emphasis on car safety) that manufacturers would sell a toxic product. But, assuming you do want to avoid it.. the easiest way would seem to be, don't get a new car. Letting it air out sure seems like it wouldn't hurt either. Friday (talk) 17:36, 19 March 2008 (UTC)[reply]
I think it's more about the regulatory bodies not allowing a product to be sold that's clearly and undeniably toxic. If a company can plausibly deny a product's harmfulness (see Crisco, Dupont), that's good enough for the feds. Vranak (talk) 20:04, 19 March 2008 (UTC)[reply]
Found a couple sources: From a few years ago, saying it might be toxic, and more recently saying it's not toxic. This is what I get for not checking the wiki first- we have a decent article on this at New car smell. Friday (talk) 17:40, 19 March 2008 (UTC)[reply]
This sounds similar to the more serious problem of "new house smell". Carpets, paint, furniture, etc., all outgas fumes, some of which are toxic if they build up beyond a given concentration. This is more of a problem in recent constructions because they are designed to be more airtight. You'd think they would start to offer a service to air rugs and furniture out before they are installed, so as to allow the toxic fumes to exit. StuRat (talk) 04:10, 20 March 2008 (UTC)[reply]

New Solar Chip

I found an interesting article and forgot to bookmark it. Now I cannot find it again.

I was looking at a new type of photovoltaic device that converts sunlight directly to electricity using a photochemical nano-gate composed of niobium [columbium], gold and silver, interchangeably, on one side of the gate. There was a separate and different word to describe this type of PV cell in its own page, and I cannot remember the word.

I have tried every combination of "solar power energy photovoltaic [columbium niobium] gold silver" that I can think of in your search engine, and in Google's search engine with wiki as the first word, and I can not find the article again.

Please help me find that page again.

The above was posted to the Help Desk recently and responses have not been helpful. Does anyone have any info on the above described chip, now undergoing testing, that was described on a Wikipedia page? External links welcomed.

was it a gratzel cell? 131.111.100.44 (talk) 18:05, 19 March 2008 (UTC)[reply]

Gravity

I was just reading an article on howstuffworks.com and i though came to me, if all the gravity on earth magically disappeared, would everything float up, surely it would just stay put, coz there wouldn't be a force acting on it?Vagery (talk) 19:20, 19 March 2008 (UTC)[reply]

Not exactly. Objects would slowly move away from the Earth and slightly east as the Earth rotated away from them. Drawing a picture of the rotating Earth and the object on it may help. It isn't that the object accelerates up, it's that the Earth underneath accelerates away from the object. anonymous6494 19:27, 19 March 2008 (UTC)[reply]
Thats what i thought would happen... just checking, thnx alot manVagery (talk) 19:29, 19 March 2008 (UTC)[reply]
Um, a person at the equator is traveling with the Earth's rotation, at a rate of 40,000km (earth's circumference) per 24 hours (length of one rotation), or, if my quick math is right, about 360m/s. Initially, said person will appear to stay put, but will in reality start moving 360m/s in a straight line, as will the ground under his feet. It wouldn't take long for things to start spreading out at that speed. Naturally, the velocity will be lower for people not on the equator, but you can do the math for that yourself, multiplying that velocity by a trig function that gives you 1 for the equator and 0 for the poles. -SandyJax (talk) 19:58, 19 March 2008 (UTC)[reply]
Would the entire Earth fly apart? The plates of the crust aren't firmly stuck together, so there'd be nothing holding them down, right? And then the layers underneath are mostly solid only because of pressure, so once the layers above are gone, they'd become fluid again and spin off? Or would they cool down fast enough to remain solid? --Allen (talk) 20:38, 19 March 2008 (UTC)[reply]
If gravity disappeared only in a sphere equal to the size of Earth, then everything within that sphere would move in the same direction relative to the distant stars that it was moving at the instant that gravity disappeared, so yes, it would all fly apart pretty quickly. There would be no more reason to rotate and everything would move in straight (geodesic) lines instead. Electromagnetic forces would still keep stuff stuck together, i.e. rocks would still be rocks and magma would quickly solidify into rocks, all those rocks would continue in a straight line. That's how I see it anyway. Franamax (talk) 21:14, 19 March 2008 (UTC)[reply]
Clarify this, I didn't mean "instead" - everything is already moving along the geodesic, it just so happens that the Earth's mass shapes the local space-time continuum so that the "shortest path" is to spin around in a circle (the shortest path is actually to the centre, but that pesky ground keeps us from getting there). Removing gravity from the sphere around Earth means that the geodesic path is now determined by all the rest of the gravity in the universe, space-time is no longer warped right here, all the marbles will roll down other slopes. That's probably not clarifying anything! Franamax (talk) 22:19, 19 March 2008 (UTC)[reply]

Battleships in space

Years ago I read somewhere that steel retrieved from pre-1945 battleships was an essential component of satellites. The reason given was that all the steel smelted since 1945 (or shortly thereafter) was contaminated with radioactive elements from nuclear explosions to sufficient degree that it would affect the satellite instrumentation.

I'm not sure whether this was only for the Pioneer/Voyager probes and I'm not sure whether it was just one particular battleship, but I know I read it once and I've never been able to find a reference since. Can anyone help me out? Franamax (talk) 20:58, 19 March 2008 (UTC)[reply]

Operation Deadlight 83.100.183.180 (talk) 21:15, 19 March 2008 (UTC)[reply]
Wow, that was fast! Thank you Wikipedia, and thank you all the anonymous editors who help to build it, especially you 83.100. I believe it was the scuttled ships at Scapa Flow that I had read about. Next question: what radiation detectors is this steel used in? All of them? Where can I read up on this some more? And does anyone have a specific reference to satellites? I'm trying to build a killer trivia question here, or maybe even a Wikipedia article. Thanks. Franamax (talk) 21:38, 19 March 2008 (UTC)[reply]
Very old lead is also used. —Preceding unsigned comment added by 58.84.80.184 (talk) 21:59, 19 March 2008 (UTC)[reply]

Wow, sorry to hijack the question, but am I understanding this correctly? All steel produced since Hiroshima/Nagasaki is tainted with radioactive elements? The Operation Deadlight article isn't very clear, and I'm having a hard time believing it's true. What mechanism causes iron mined on, say, the other side of Japan to become contaminated? Thanks, Icthyos (talk) 11:03, 20 March 2008 (UTC)[reply]

I would guess (but have not been able to verify) that it is the oxygen blown through during the conversion process (see Linz-Donawitz process and Bessemer process) that has atmospheric contamination, not the ore. -- Q Chris (talk) 12:41, 20 March 2008 (UTC)[reply]
Yes, it is the air used smelting the iron and oxygen used in making the steel that introduces atmospheric contamination. Old steel can be hot-reworked so that no new air is incorporated. Incidentally, people born since 1945 can have their age determined (plus-minus a few years) by the radioactivity of their teeth. Franamax (talk) 17:12, 20 March 2008 (UTC)[reply]
It isn't just Hiroshima and Nagasaki—it's the other ~600 or so atmospheric tests as well that took place from 1946-1980 by various nations in the world. It'd be interesting to total up the cumulative megatonnage from the years of atmospheric testing, but it is well over 100MT. It would be wonderful if someone had data for background radiation levels over the 20th century, I'm sure it would be an interesting graph. --Captain Ref Desk (talk) 19:33, 20 March 2008 (UTC)[reply]
Nature Vol.437 p.433 15Sep05 Age written in teeth by nuclear tests Spalding et al has an interesting graph of atmospheric carbon-14 levels. I have a Nature subscription, see if you have access to the article here. There is some info on copyright licensing here which I don't understand, but I doubt it would be released under GFDL. The C14 ratio is stable until 1955 (when above-ground testing really got going), rises quickly to +20% by 1960, dipsy-doodles 'til '62, then skyrockets to +85% by 1965, exponential decay thereafter. We're down to approx. +8% now. Other radiation levels would likely look similar as it is diffusion out of the atmosphere that predominates over radioactive decay. Of course, a lot of the radioactive elements would still be in organic materials, water, teeth, etc. Franamax (talk) 20:00, 20 March 2008 (UTC)[reply]
PS There are little notches at 1972 and 1980 where further tests had been conducted. Franamax (talk) 20:04, 20 March 2008 (UTC)[reply]
That's not bad. Not quite as ideal as having the direct background radiation data, but still not bad. I looked at the supplementary data to see where their data source for the background C14 levels were from and found a few useful references... will see if there's data I can make a new graph out of. Supposedly all the data from that issue should be at this site but the site is current down. :-( --Captain Ref Desk (talk) 01:19, 21 March 2008 (UTC)[reply]
Actually, now that I look at it, I've seen these graphs before. We have one not too dissimilar even though it's only from the Southern Hemisphere. The data sets above are much more complete (judging from the articles in Radiocarbon) so maybe we can make a more exciting one in the near future. --Captain Ref Desk (talk) 01:24, 21 March 2008 (UTC)[reply]
Nice find on the Commons image! According to Greenpeace, 711 atmos or water tests, 438 Mt total atmos test yield, 4200 kg Pu atmospheric discharge(!!) - Greenpeace of course being a neutral source :) (though there's no reason to think they wouldn't be reliable on this).
I couldn't get at the Radiocarbon journal data, unless Jimbo wants to buy us those issues. I agree that total release of radionuclides and overall radio-persistence would be interesting. Now include decay products, organic persistence, half-lives - add in contaminated steel, dating of teeth, and one more persistent effect we could identify - we've got ourselves a new Wikipedia article: Persistent effects of atmospheric nuclear detonations. What's your diagnosis Doctor Strangelove? Franamax (talk) 02:01, 21 March 2008 (UTC)[reply]

Wiki Myth Busters...

So when you pull the ol' snap your friend in the butt with a towel prank, the snapping noise is caused by the end of the towel breaking the sound barrier. True or false? Beekone (talk) 21:09, 19 March 2008 (UTC)[reply]

Why wouldn't the snapping noise just come from part of the towel moving quickly? Mac Davis (talk) 21:50, 19 March 2008 (UTC)[reply]
According to Whipcracking, at least some whips create a cracking noise by breaking the sound barrier. Whether the towel works that way I don't know; it seems a lot shorter than most of the whips shown as being crackable in the article. --Allen (talk) 21:57, 19 March 2008 (UTC)[reply]
Straight Dope talks about it: http://www.straightdope.com/mailbag/mtowelsnap.html DMacks (talk) 22:05, 19 March 2008 (UTC)[reply]
Seems highly unlikely to me. You can 'snap' a towel (wet or dry) pretty slowly and still get a pleasing noise. The towel changing direction is large enough to compress a sufficient amount of air to create a fairly flat noise. And any ol' Joe can do it pretty much first time every time. The tip, or any other part of the towel, doesn't have to moving anywhere near the speed of sound for this to occur. This is different to cracking a whip where a long whip allows a small tip to achieve a very high speed, greater than the speed of sound, resulting in the loud crack, and it's rather more difficult for people to do without some practice. Move a whip slowly and it won't crack at all. Personally I'd say 'Busted', or at least 'Implausible'. --jjron (talk) 14:03, 21 March 2008 (UTC)[reply]

medical terminology, not sure of spelling or existence

I was told to go to "fae" under frequent asked question. I'm trying to find out about physcadose. Possibly something to do with the liver. Is there such a word? I'm not sure of the spelling, can you help me?71.91.46.235 (talk) 22:20, 19 March 2008 (UTC)[reply]

Definitely not a word as you've written it, and no similar sounding word pops to mind. Where did you hear this word? - Nunh-huh 23:37, 19 March 2008 (UTC)[reply]
It's a bit of a stretch, but sarcoidosis? -- MacAddct  1984 (talk • contribs) 01:43, 20 March 2008 (UTC)[reply]

Turtle identification

I recently took photos (here is one of them) of a turtle on our pond that seems to be a Chicken Turtle (thanks to Joelr31 for pointing me in the right direction). I wanted to use a cropped version of the previously linked image in the chicken turtle article, but first I'd like to confirm the identification here. After looking at a bunch of pictures on the web, I'm not perfectly sure it is, in fact, a chicken turtle. As for geographic info, I live in upstate South Carolina, USA. I don't really have ideas of what else it could be, but some of the coloration/shell ridges of this turtle don't quite seem like the species, although there's probably room for variation. Are there any other species it could possibly be, or is it pretty obvious that it's a chicken turtle? Just wanted to make sure of this to hopefully prevent some misinformation here on Wikipedia. :) --JamieS93 23:41, 19 March 2008 (UTC)[reply]

Well, it could easily be the more common Painted Turtle, or something else entirely. The red coloration doesn't appear to be standard for a Chicken Turtle, and while his/her neck is long, it isn't as long as some pictures identified as chicken turtles. Personally I'd be hesitant, but I don't know a thing about turtles.--Captain Ref Desk (talk) 20:15, 21 March 2008 (UTC)[reply]

March 20

Capacitor-Batteries

Can capacitors be controlled to discharge at a near constant voltage using some electrical circuitry apparatus? In other words, I'm asking if a capacitor can be used as a battery, and how a circuit could be/is constructed to do this.

Thanks, 74.173.90.59 (talk) 00:28, 20 March 2008 (UTC) Sam[reply]

I think a capacitor that works like a battery is a battery. See battery capacity and discharging -- MacAddct  1984 (talk • contribs) 02:01, 20 March 2008 (UTC)[reply]

Ultracapacitors with carbon nanotubes covering plate surfaces to increase surface area may in the future replace common dry-cell batteries. Mac Davis (talk) 03:16, 20 March 2008 (UTC)[reply]

I think you two are totally missing the point of this question. The voltage of a battery falls off slowly as the remaining charge decreases, until the end when it falls off sharply; e.g. it could have half a charge but still 90% of the original voltage. The voltage of a capacitor, on the other hand, always falls off linearly with the remaining charge, because the energy is stored in the form of an electric field produced by separated charges. Sam here is asking whether it's possible to modify the charge-voltage relationship of a capacitor to seem more like that of a battery by using an electric circuit. I don't know the answer; I just wanted to clarify what the question is. —Keenan Pepper 05:02, 20 March 2008 (UTC)[reply]
You can use a DC-DC converter. It can take a non fixed voltage and convert it to a fixed output voltage. There will be a lower limit where the input will fail to be enough to drive the circuit. These can also be found in cameras and LED torches to extend the life of the battery. Graeme Bartlett (talk) 05:10, 20 March 2008 (UTC)[reply]

Just got back from the doctor who told me something that has left me in a bit of shock. In America where there are ample medications to treat TB vaccines are not given as in some parts of Europe, Asia and Africa where the reverse is the case. The reason is that once a person is immunized against TB they develop antibodies, the problem being that the test for TB is based on whether or not a person has the antibodies. In America people are not immunized so they can serve as indicators of an epidemic. Does this mean that in America as far as the government or health care system is concerned the it is better that the population serve as human Guinea pigs than to be immunized against a disease despite the fact that almost no one in America would be immunized against TB in the event of a major catastrophe like nuclear war, followed by bombardment with biological weapons? — Preceding unsigned comment added by 71.100.10.177 (talkcontribs)

According to the CDC, the "BCG vaccination is not generally recommended in the United States because of the low risk of infection with M. tuberculosis, the variable effectiveness of the BCG vaccine against pulmonary TB, and the vaccine’s interference with the ability to determine tuberculin reactivity."[13] table of contents. -- MacAddct  1984 (talk • contribs) 03:18, 20 March 2008 (UTC)[reply]
That would certainly confirm my doctor's comments and my fear regarding a situation where in the absence of vaccination contraction of the disease by a major portion of the US population would be too late to prevent. — Preceding unsigned comment added by 71.100.10.177 (talkcontribs)
I think basically the CDC has decided (and weighed the +/-) that the vaccine in adults is nowhere near effective enough and would just rather have doctors know if someone is infected or not. Otherwise we'd all have to get chest X-rays every year. Also there were only 646 reported deaths from TB in the US in 2005 out of 14,200+ people infected. I think a smallpox outbreak is something more more disconcerting. [14] -- MacAddct  1984 (talk • contribs) 05:13, 20 March 2008 (UTC)[reply]
Just to correct a statement by the original questioner, it's not a question of antibodies. Someone who has received BCG vaccine, or been exposed to tuberculosis, develops cellular immunity, which (rather than humoral immunity) is the mechanism responsible for positive PPD reactions. As to the other point, TB doesn't spread quickly enough to be an effective biological weapon. - Nunh-huh 06:37, 20 March 2008 (UTC)[reply]
Thanks for the clarification. However, in a situation unlike in Japan where there were intact resources surrounding the two bombings that would not be the case following a nuclear exchange today after which one might expect a combination of biological weapons meant both to kill and to exhaust resources. In other words TB might not be an effective killer but it might be one heck of a resources user along with a mired of similar lack luster bio weapons. America has been caught with its pants down before.
What can I say. The experts differ with your assessment. In a situation where healthcare resources are strained or absent, problems arising from sanitation, such as cholera and dysentery, and other diseases, they feel, are more likely to cause immediate problems than TB. The downside of BCG vaccine (ineffectiveness, and rendering tuberculin tests far less useful) are felt to outweigh the advantages (some level of immunity in part of the population). - Nunh-huh 08:37, 20 March 2008 (UTC)[reply]
United States civil defense has always been very weak. It tends to remind the voters how nasty these sorts of weapons are. --Sean 14:12, 20 March 2008 (UTC)[reply]

Worrying about whether TB would be a problem in the wake of a nuclear exchange seems, to me, to be akin to worrying about how the problem of racial prejudice would be affected after an impact event. Yes, we can dream up all sorts of mad scientist situations but if there isn't a good reason to believe them plausible (or the most effective way for anyone to get what they want) it's not worth the resources to worry about them, much less act upon those resources. There are far more effective agents for use as biological weapons than TB; if we are talking about an enemy that can start nuclear war (and would?), I'm sure they could do far worse. --Captain Ref Desk (talk) 16:57, 20 March 2008 (UTC)[reply]

Foxes in the UK

Isn't it strange that you can find foxes in Glasgow and London but not in Newcastle?Mr.K. (talk) 03:26, 20 March 2008 (UTC)[reply]

It would be strange. Do you have any references to this? -- Q Chris (talk) 12:20, 20 March 2008 (UTC)[reply]
Not more than personal opinion. Mr.K. (talk) 17:38, 20 March 2008 (UTC)[reply]
Mange perhaps? Some years ago I remember the foxes in Bristol almost died out from mange. I think outbreaks of mange remain localised.--80.176.225.249 (talk) 18:51, 20 March 2008 (UTC)[reply]
Maybe the upper class prefer to hunt in Newcastle? Nil Einne (talk) 12:06, 21 March 2008 (UTC)[reply]

sound transducer

how can i make a sound transducer by myself in home?actually i want to make a switch which can be turned on by the sound of clapping .

have a look at [15] or other links from a search for "clapper switch circuit", with which i found the given url. sorry no-one answered your simple request earlier. 153.1.253.5 (talk) 13:40, 20 March 2008 (UTC)[reply]

Quantum Mechanics: Orthogonality of Dirac Delta Function

The functions and are said to be orthogonal on interval if their inner product is zero

(1)

For complex-valued functions or kets and , they are said to be orthogonal on interval when

(2)

To continue my last discusstion, Quantum Mechanics: Entangled Wave Function, my question is how to prove the orthogonality of the Dirac delta function mathematically? Or some related resource? Thanks! - Justin545 (talk) 08:53, 20 March 2008 (UTC)[reply]

This is really a maths question, but the way to prove it is to look at the definition. The dirac delta function is zero at all the places except for the argument. If you multiply this zero value by another dirac delta you, will get a zero, unless the arguments are the same, when it will be greater than zero. if x not equal y. Graeme Bartlett. Another way to look at the dirac delta is that it is a sampling function, when you integrate its product with another function, it samples the other function at the argument to the dirac delta. (talk) 11:14, 20 March 2008 (UTC)[reply]

VHF/FM Radio Frequencies

The current commercial FM waveband is 88-108 MHz. However, I am trying to discover exactly when this spread came about.

I have a number of radio receivers, the dials on each differing significantly. One, an American set, covers 90-108Mhz. Two others, both British, cover 88-101 and 88-104 Mhz respectively. I have, however, been unable to ascertain when the full spectrum (88-108) came into use. Can anybody help?

Samilong (talk) 10:29, 20 March 2008 (UTC)samilong[reply]

Take a look at FM band, it does not answer your question, but shows even more alternatives. Graeme Bartlett (talk) 11:49, 20 March 2008 (UTC)[reply]
FM broadcasting in the USA looks like it covers more of the specifics, though it doesn't date the extension of the range from 106 MHz to 108 MHz. — Lomn 12:53, 20 March 2008 (UTC)[reply]

spin and (hyper)complex numbers -- why is the quanta of angular momentum 1/2

What is the connection between the intrinsic spin of quote-unquote elementary particles and the imaginary operator(s) defining complex numbers (and their further generalisations)? Intuitively, it seems they ought to be reasonably closely related, but this does not appear to be reflected in the "popular" literature. (One supposes that this intuitive sense could be ellaborated upon, if really necessary, but does not feel that it ought, as the connection is expected to be known, if not superficially evident.) 153.1.253.5 (talk) 13:53, 20 March 2008 (UTC)[reply]

I don't think there is any intrinsic connection.
One reason why spins are often defined as 1/2 or -1/2 is that the spin difference between the two states is one . 1. A nice simple number and ideal for quantisation. Quantisation means allowing a property to have only integer multiples of a value. Eg if energy can be 10,20,30,etc (and not 22.3, 35, or 44) Joules then that energy is said to be quantised. Quantisation can also refer to methods that treat non-quantised properties as being quantised for the purposes of simplicity (eg when contructing a model or theory)
Spin is assumed to be intrinsically quatised; why that is so is another question.83.100.183.180 (talk) 18:17, 20 March 2008 (UTC)[reply]
The reason imaginary operators/complex numbers were not used is simply that they were not needed to describe the situation. Negative number possess sufficient weirdness to do the job. Have a look at Quaternion for a 4d consistent system, (IMHO) Its only a matter of time before this is used to describe a physical processes. You might find this interesting from a quantum point of view Spin-statistics theorem. GameKeeper (talk) 21:34, 20 March 2008 (UTC)[reply]
Quaternions are used to represent rotations in 3D and 4D space - see Quaternions and spatial rotation. Indeed, part of William Rowan Hamilton's reasoning that lead to his discovery of quaternions was a search for a higher dimensional analogue of the way on which 2D rotations can be represented as multiplication by complex numbers with unit magnitude. Gandalf61 (talk) 10:46, 21 March 2008 (UTC)[reply]

Tapdancing gulls

We haven't had a gull themed question in a while so here goes. Some local species of gull tap their feet onto the ground. My guess is it's either some strange courtship ritual or perhaps a method to draw insects out of the ground (I only see them do this on grassy fields). What do you guys think? PvT (talk) 14:11, 20 March 2008 (UTC)[reply]

It makes worms come up. This answer says the sound annoys the worms, although I've heard other explanations that have it that the worms think the sound is rainfall and come up for a drinky. -- Finlay McWalter | Talk 14:18, 20 March 2008 (UTC)[reply]
I thought worms got all the moisture that they need from the ground and the only reason they come above ground when it rains is so that they don't drown. Dismas|(talk) 15:06, 20 March 2008 (UTC)[reply]
Does it look a bit like they're doing an Irish jig? if so, yes - they're trying to get the worms to come to the surface. There's a good video of several different gulls 'rain dancing' here. I've often wondered how it is that the gulls know exactly where they need to 'dance' in order to bring up a worm. There definitely seems to be an element of skill to it - they'll wander back and forth with their heads cocked, trying to find just the right spot. Are gulls' ears sensitive enough to hear the worms moving beneath the surface? Maybe they're just looking for fresh worm casts (I personally wouldn't put it beyond a gull's intelligence to know what a worm cast is...)? --Kurt Shaped Box (talk) 18:46, 20 March 2008 (UTC)[reply]
I'd expect that any decent soil (as opposed to sand, clay, or rock) would contain worms, since they are common enough that I don't see a need to search for individual worms. I now have the urge to sneak up on a sleeping gull and glue on a pair of tap shoes, to make the dance even more entertaining. StuRat (talk) 19:12, 20 March 2008 (UTC)[reply]
If you think watching gulls do it is entertaining, try watching flamingos. Daniel (‽) 20:33, 20 March 2008 (UTC)[reply]
It's even funnier to see a young gull (it's always the recently-fledged ones I've seen doing it) determinedly dancing for worms on concrete or tarmac. I even saw one doing it on a flat rooftop once. Not *quite* as funny as seeing a woodpecker attempting to drill a steel lamppost - but still... ;) --Kurt Shaped Box (talk) 21:15, 20 March 2008 (UTC)[reply]
Thanks for the answer guys. I'm wondering though. How do gulls learn this behaviour? Is it instinctive? Is the dance itself instinctive? Obviously they don't know by themselves that their trick won't work on solid surfaces, there's some trial and error involved. This would be an interesting research project. Sure beats finding out how far penguins can poop...PvT (talk) 11:09, 21 March 2008 (UTC)[reply]

What the gulls are doing is something like (or actually is) worm charming and it's likely passed on down the generations carefully from gull to gull so that this skill won't die out. Julia Rossi (talk) 11:52, 21 March 2008 (UTC)[reply]

That other famous gull hunting skill, namely cracking open bivalves and crustaceans by flying up and dropping them onto a hard surface is (IIRC - no source to hand, sorry) learned by young gulls from observing their parents/older members of the flock then perfected through trial and error. I'd imagine that it's a similar thing with the worm-charming. Gulls have a long childhood/adolescence - going from meek, timid, dopey and needy to the precise opposite of all those things in four years or so. They learn and grow as individuals. --Kurt Shaped Box (talk) 20:38, 21 March 2008 (UTC)[reply]

Hallucinations making everything appear very big or very small

I'm vagually sure I remember reading about this phenomenon (see subject line) a while ago, but I can't find any references to it now. Is this a recognized type of hallucination? Is there a word for it? How common is it? --86.135.178.19 (talk) 20:40, 20 March 2008 (UTC)[reply]

Micropsia or macropsia, depending. - Nunh-huh 01:35, 21 March 2008 (UTC)[reply]

Radioactive decay

How half-life work on individual atoms? If I have 8 atoms of substance X, which has a half life of 5s, then after 5s, I obviously have 4 atoms of X, plus whatever it decayed into and some radiation, right? So what if I have 1 atom of X? Is there a 50% chance that it will decay after 5s? If so, is that true for each atom in my 8 atom sample, i.e. that after 5s, there is a remote possibility that all of the atoms will have decayed, or that none of them will have? I'm very confused. 72.155.207.33 (talk) 22:31, 20 March 2008 (UTC)[reply]

Half-life, especially as it relates to radioactive decay describes a statistical probability for a "large" sample. The two articles I linked have more information about what "really " happens, especially for small collections of decaying particles. DMacks (talk) 22:54, 20 March 2008 (UTC)[reply]
To answer the specific example, your last statement is correct. With x atoms after one half-life, the most likely number of atoms remaining is x/2, though any number from 0 to x is possible. Additionally, in virtually all cases, x/2 is not probable -- that is, the odds are usually better than 50/50 that there are not x/2 atoms remaining. The various probabilities can be expressed via the combination function. — Lomn 23:51, 20 March 2008 (UTC)[reply]

So what determines what amount decays? 72.155.207.33 (talk) 04:52, 21 March 2008 (UTC)[reply]

Pure chance, as far as we understand it. Every atom has a 50% chance of decaying within one half life. That means that the probability for k out of n atoms decaying within one half life is . If you are unfamiliar with this mathematical notation, see Binomial coefficient. The expectancy is n/2, the standard deviation is . With 8 atoms, the standard deviation is thus about 1.4. The individual probabilities are:
0 atoms will decay: 1/256
1: 8/256
2: 28/256
3: 56/256
4: 70/256
5: 56/256
6: 28/256
7: 8/256
8: 1/256
Icek (talk) 06:45, 21 March 2008 (UTC)[reply]

Note also that for radioactive decay, you are in the "spooky" "God ... play[s] dice with the universe" world of quantum mechanics, where you pretty much have to leave your intuition at the door. (In actuality, with one radioactive atom you can have the situation where it is both decayed and not decayed simultaneously. Just ask Schrödinger's cat.) -- 128.104.112.85 (talk) 00:49, 22 March 2008 (UTC)[reply]

Wow, that's weird! Thank's for the link. Is this actual science, or is it an If a tree falls in a forest and no one is there to hear it sort of philosophical thing? 72.155.207.33 (talk) 01:01, 22 March 2008 (UTC)[reply]
That quantum mechanics is generally not as simple as "if no one hears the tree falling" can be seen by Bell's theorem for entangled particles. Icek (talk) 07:08, 22 March 2008 (UTC)[reply]

scientific name

what is the scientific name for fucking the shit out of someone excuse my french (yes i mean sexually)

Coitus (wiktionary) is a commonly-accepted term. — Lomn 23:52, 20 March 2008 (UTC)[reply]
Anal sex might fit the questioner's term more closely. Edison (talk) 23:55, 20 March 2008 (UTC)[reply]
There is no scientific term for unusually hard sex, if that's what you're asking. Mac Davis (talk) 00:10, 21 March 2008 (UTC)[reply]
But hey, let's make one up! How about Coitus maximus? (Someone who knows Latin could probably do better)--Captain Ref Desk (talk) 01:07, 21 March 2008 (UTC)[reply]
How about Spanish instead ? "cópula hasta muerte". StuRat (talk) 02:05, 21 March 2008 (UTC)[reply]
"Coger hasta que no haya mierde en el cuerpo" (only in latin america)

March 21

Free falling human body

Let's say that a person is very high up in a building (the 100th floor, 200th floor, whatever). And he falls / jumps / is pushed out of the window and falls to the ground. (No parachute, no nothing.) I have heard that the human body would be dead before it hits the ground. Is that possibly true? In other words, would you die while in mid-air (and of what, exactly?) or does the impact at the ground kill you? When people were falling / jumping from the Twin Towers on 9/11, I remember people saying that they would have been dead before hitting the ground. Shock? Heart attack? Pressure/force of gravity? So, what's the deal with all this? Thanks. (Joseph A. Spadaro (talk) 07:12, 21 March 2008 (UTC))[reply]

This idea is a myth, based on poor scientific knowledge and limited reasoning powers, possibly propagated to help the victims' relatives deal with the loss more comfortably. There are many instances of people falling from great heights, for example from flying planes, and surviving.[16] There is also a worldwide sport of deliberate freefalling from planes at high altitude, the participants seem to be able to survive this experience with renewed vigour and an urge to repeat the process. Like the old saying goes 'It ain't the fall that kills ya, it's the stop at the end that does it'Richard Avery (talk) 08:24, 21 March 2008 (UTC)[reply]
BTW, according to this [17] (not the best source but I'm sure someone here could calculate it, heck I probably could but I'm lazy) terminal velocity will usually be achieved from a 96 floor jump. Anything higher is therefore irrelevant except it takes you longer to hit the ground. Indeed the terminal velocity article raises another important point. The people who jump off high buildings probably don't even go anywhere nearly as fast as free fall divers since free fall skydivers optimise their body position and clothing to reduce air friction i.e. increase their terminal velocity. People jumping off buildings don't, they may not even have the time to do so that well. Nil Einne (talk) 12:01, 21 March 2008 (UTC)[reply]
...free fall skydivers optimise their body position and clothing to reduce air friction i.e. increase their terminal velocity.
Are you sure about that? I thought the standard skydiving position (back arched, pelvis thrust forward) was designed to keep you facing down instead of up at the sky. If you wanted to fall faster, I think you would do it by either curling into a ball and falling buttocks-first, or maybe pointing your arms above your head and falling hands-first. The pelvis-first position doesn't seem particularly optimized for speed. —Keenan Pepper 15:48, 21 March 2008 (UTC)[reply]
Like Einstein's Brain, they saved Kittinger's Testicles.
Indeed. This fascinating source says that terminal velocity in normal skydiving position is about 125 mph, but adopting a "balled up" position increases this to about 200 mph. The free fall speed record without special equipment is 321 mph, and Joseph Kittinger reached speeds of over 600 mph in his record-breaking parachute descent in Project Excelsior. Gandalf61 (talk) 16:53, 21 March 2008 (UTC)[reply]
A person with a weak heart can, of course, have a heart attack while in suspense about the immediately outcome of hitting the ground. Gravitational acceleration is unlikely to kill because it must be less than one g, half the force delivered on some roller coasters. As for air pressure, at terminal velocity, the force applied by the air is equal to the force applied by gravity; that's why the velocity remains constant. An adult human weighs approximately 75 kg and has a cross-section of about 30 cm x 170 cm, so that's 15 g/cm^2 of pressure. Decidedly not a lot. A person diving head-down would experience 330 g/cm^2 of air pressure, with the reasonable assumption that the head's cross-section is 15 cm x 15 cm and the unreasonable assumption that all air resistance is provided by the head. 330 g/cm^2 is a lot, yes, but I don't think it's enough to kill.
Note that g is a measure of mass, not force or weight, so expressing pressure as g/cm^2 is technically incorrect. I did it to make visualizing the pressures easier. --Bowlhover (talk) 18:16, 21 March 2008 (UTC)[reply]

So, basically ... you are alive up until the very moment of impact? And what exactly causes the death at that point? Thanks. (Joseph A. Spadaro (talk) 05:25, 22 March 2008 (UTC))[reply]

The Project Excelsior article notes that the reason for Kittinger's high-altitude test jumps was that Air Force scientists were worried that after an ejection at high altitude, a pilot might go into a rapidly rotating fall, at speeds of up to 200 rpm, which could be fatal (I guess they observed this with test dummies) I don't konw if that is a real concern for a person, but that is one potential way in which a person might die before he or she hit the ground. --Bmk (talk) 07:34, 22 March 2008 (UTC)[reply]
Joseph, are you serious? You can take your choice depending on which part of the body hits the ground first. Perhaps a crushed skull with subsequent brain trauma? ruptured heart and major blood vessels? Broken neck and severing of spinal cord? Landing feet first can thrust the femurs up into the chest cavity with consequent damage to the cardiovascular system. I'm sure there are several other serious trauma scenarios that ultimately cause death. Richard Avery (talk) 07:34, 22 March 2008 (UTC)[reply]

amplifier

why amplifier have high input resistanceNippun makkar (talk) 07:34, 21 March 2008 (UTC)[reply]

Well, the full answer is that they do not always have a high input impedance. Those that do are designed that way so that the input voltage is not dropped by the input resistance of the amplifier. For instance, if the input impedance was equal to the impedance of the source voltage then the voltage at the amplifier input terminals would be exactly half the sources unloaded voltage. But surprisingly, this might be desirable in some cases. To avoid signal reflection at the input, the amplifier input impedance needs to match the source as closely as possible. In RF applications, amplifiers are often designed in the common base configuration as this results in an input impedance close to the 50Ω characteristic impedance commonly used for RF transmission lines. It can also be desirable to have an input impedance of less than infinity for noise reduction reasons. For the kind of audio frequency amps you are probably referring to, where high input and low output impedances are required, then common emitter is the configuration to use for signal level amps or some kind of push-pull arrangement for power stages. SpinningSpark 09:55, 21 March 2008 (UTC)[reply]

Specific Heat Capacity

In an experiment to find the specific heat capacity of a metal, it is found that 5200 J is needed to raise the temperature of a 2 kg block by 20 oC. How to find the specific heat capacity?

Have you looked at our specific heat article. From that you will see that specific heat is measured in joules per kg per . Do you see how to calculate it now? SpinningSpark 09:35, 21 March 2008 (UTC)[reply]

Heat capacity is Joules per kgram per degree C.
So you need to work out how many joules to raise a block of 1kg the temperaure 1 degree C
eg if it take 100 J to raise the block 3C then in takes 100/3 J to raise it one.
Can you take it from there. (ask again if you get stuck)87.102.16.238 (talk) 12:02, 21 March 2008 (UTC)[reply]

Modeling Friction

Friction is usually modeled by a constant multiplied by a normal force. This is the model used in the friction article, and the one they taught me in high-school physics. It is quite clearly wrong. We all know there's a reason drag racers have big wheels, but with that model of friction it makes no difference. In my robotics class, we once made "robots" go up a slope. They held better when more weight was added. It should be the same. Is it just too complex to make a better model? — DanielLC 16:50, 21 March 2008 (UTC)[reply]

Well, for drag racers, the large rear wheels aren't to add weight but to increase surface area and give enough thickness to allow some of the rubber to melt, increasing friction further. This is why you see other auto racers weaving back in forth to melt their tires. -- MacAddct  1984 (talk • contribs) 17:06, 21 March 2008 (UTC)[reply]
There's some discussion of the less-than-ideal nature of friction and the ideas taught in school here. I would suspect that there are so many variables, an overall model is too complex for most uses, but you could probably create a good, useful model for any particular recurrent situation. So you could probably create a useful model for your robots, assuming you don't vary much between trials, but that model probably wouldn't hold for other situations. But I am not a friction expert :) Skittle (talk) 17:14, 21 March 2008 (UTC)[reply]
To give credit where it is due to WP, our coefficient of friction article does say that the COF is a system property, rather than a property of the pure materials. As Skittle says, once you have set up your system and chosen the right value of μ, then the simple linear equation is a reasonably good approximation. If you start adding weights and deforming your tyres or creating adhesion, then the equation will cease to apply. That doesn't make it "wrong". --Heron (talk) 10:29, 22 March 2008 (UTC)[reply]

universe

is there an end to the universe? either way how do you know. — Preceding unsigned comment added by 72.155.39.82 (talkcontribs)

Current theory says yes, and it is ever expanding. See observable universe -- MacAddct  1984 (talk • contribs) 17:14, 21 March 2008 (UTC)[reply]
Another answer would be to say, no, there is no end to the universe, but it is "closed" - if you and a buddy head off in exactly opposite directions, you'll eventually meet up again. From Nature journal: "If the Universe is closed, and has a small enough diameter, we may be able to see right round it because photons can traverse the whole Universe" and "The WMAP data ... suggest that we might indeed live in such a small closed universe". Of course, I'm not all that good at reading this stuff :) Franamax (talk) 20:54, 21 March 2008 (UTC)[reply]

Boomerang returns in space. Make 'splain that one now!

There have been some posts floating about socialbookmarksville relating to a japanese cosmologist's demonstration that a boomerang returns in space. Unfortunately, however, the articles have all been rather sparse on content, and one is at a loss to find any discussion on the matter. The obvious question that arises is: what's goin' on there, then? From my admittedly limited memory, all explanations of boomerang dynamics have involved recount to aerodynamics of the 'leading edge is moving faster through the atmosphere, and so generates more lift' ilk. Having always been rather dubious of the canonical explanations of "wot makes dem aerofoils werk" (planes fly upside down you know, if WWI movies have taught me anything), ones interest is perked as to what possible mechanisms might be put forward for this interesting result. Chards of regards, 83.102.28.140 (talk) 18:30, 21 March 2008 (UTC)[reply]

I see no reason why a boomerang would work in space. Boomerangs work by use of an airfoil and lift both of which wouldn't work in a vacuum. Otherwise our spaceships could take off and fly around with wings. -- MacAddct  1984 (talk • contribs) 18:42, 21 March 2008 (UTC)[reply]
Japanese astronaut Takao Doi did not throw the boomerang in space as such. He threw it inside the pressurised International Space Station and thus had all the benefits of aerodynamics to assist its flight. SpinningSpark 18:48, 21 March 2008 (UTC)[reply]
Ah, that makes more sense. No gravity, but there is an atmosphere. -- MacAddct  1984 (talk • contribs) 18:53, 21 March 2008 (UTC)[reply]
I would question the wisdom, though, of deploying a hunting weapon on board a spaceship. SpinningSpark 19:48, 21 March 2008 (UTC)[reply]
Most commercially available boomerangs aren't useful as hunting weapons. Making them reliably aerodynamic, so they come back, destroys their ability to deal any significant amount of damage. Hunting boomerangs I've seen usually have one end blunted and weighted, making them fly straight and hit hard. A proper throw can make them curve through the air, which probably led to experimentation in design and the more modern-looking boomerang. -- Kesh (talk) 00:32, 22 March 2008 (UTC)[reply]
Thinking about it, what would be the point in the first place of having a throwing weapon that returns to the user - if it's only going to be used for hunting animals (I suppose that such a device might have its uses on the battlefield)? If you throw and miss, your prey has more than likely seen your attempt and decided to bolt - whilst you stand there waiting for the thing to come back. It would make far more sense to carry a few 'one way' boomerangs and whang them one after the other in the path of your quarry's flight in rapid succession. --Kurt Shaped Box (talk) 00:52, 22 March 2008 (UTC)[reply]
There were two reasons why this was important, both stemming from weapons being expensive (in terms of time invested in making them) back when they were hand-made. Thus, you might not have very many, so didn't want to lose or damage them by having them land who-knows-where. Also, it would take quite a bit of time to track down thrown weapons even if they could be retrieved intact. This is time that could be better spent hunting. StuRat (talk) 03:33, 22 March 2008 (UTC)[reply]
Wouldn't the stray hydrogen atoms in the vacuum of space *eventually* have an effect on the boomerang's flight? --Kurt Shaped Box (talk) 20:59, 21 March 2008 (UTC)[reply]
I would expect stray rocks and the gravity of large objects to have an effect first. -- kainaw 21:11, 21 March 2008 (UTC)[reply]
Darn, boring old reality strikes again. 83.102.28.140 (talk) 03:34, 22 March 2008 (UTC) (OP)[reply]
No reason to let boring reality get in the way of answering an interesting question. This gives the vacuum pressure of space as 10 pPa. That is a factor 1016 lower than earth sea-level. Lets say a typical boomerang throw on earth is aimed at a target a distance of 50 feet away and the boomerang travels in a circular path. Distance travelled by the boomerang to return is 300 feet (pi is 3 in my universe). The same throw in space will have 1016 less "lift" in the direction of curve and consequently the circumference of the curved path is 3 x 1018 feet or 1018 metres (there are 3 feet to the metre in my universe). If you threw the boomerang at a velocity of 20 m/s it will return to you in about 88 million years if you manage to miss all the kangaroos and other significant objects in the universe. SpinningSpark 09:47, 22 March 2008 (UTC)[reply]
More of an unrelated history answer but..as stated above most boomerangs were the non returning kind that would be used for hunting (mainly) kangaroos and wallabies. The returning kind would be thrown over a flock of birds on a body of water, with the intent (I think) of getting the birds to get scared and fly over the ground where other boomerangs can be thrown at them.--Shniken1 (talk) 12:20, 22 March 2008 (UTC)[reply]

Probably a FAQ but couldn't find what I am looking for!

The metric expansion of space over time is clearly not equivalent to applying an enlargement transformation to all the objects in the universe by a certain scale factor. For example, it doesn't "stretch" the solar system, or pull galaxies apart, or on a smaller scale, tear apart the atoms in molecules (a context where the primary force is not gravitational). However, the distance between distant galaxies does increase.

The intuitive explanation of why in the examples I gave where the distance does not actually increase, is that they were being held together by forces that prevented separation. Distant galaxies are not bound in the same way. Is this intuitive explanation reasonable? Or is it a mistake to view changes to the metric as in some way analogous to the application of a force? IBangMyHead (talk) 19:20, 21 March 2008 (UTC)[reply]

On a small scale, whatever the metric of the universe as a whole, space is Euclidean to all intents and purposes. Atoms remain the same size despite the expanding space. Things made out of atoms, like trucks for instance, also remain the same size. Distances between galaxies however are increasing. Think of pieces of broken ice moving apart on an expanding ocean. The ocean is expanding, the distance between the pieces of ice is increasing, but nothing much happens to each individual iceberg. SpinningSpark 19:41, 21 March 2008 (UTC)[reply]
Yes - but my question is why clusters of matter that make up the icebergs (stay pretty coherent) behave in a different way to the clusters of icebergs (which spread out). A criticism of the "rubber sheet" analogy is that if you did paint a little picture of the solar system on the sheet as you stretched it, the planets would spread out (and indeed, the planets themselves would enlarge!).
Is it because at a local level, the effects of metric expansion are effectively impossible to observe (the icebergs do expand, but it's harder to observe any change in the icebergs compared to the distances between distant icebergs which clearly grow) or is it because forces that hold the icebergs together counteract the inflationary pressure? IBangMyHead (talk) 19:48, 21 March 2008 (UTC)[reply]
User:BenRG is usually good for these kinds of question but he does not seem to have been active on the Reference Desk for a few days. SpinningSpark 19:53, 21 March 2008 (UTC)[reply]
As I understand it, it has to do with the different distances on which the forces operate most effectively. Gravity is very weak on small scales compared to electromagnetic forces. Metric expansion of space is only going to be very noticeable on massive scales; on smaller scales, the more locally powerful forces are going to reign supreme. The structure of atoms themselves are regulated by nuclear force, which metric expansion of space couldn't affect even if it wanted to, and the structural integrity of molecules is going to preserved by the electromagetic force. Does that explain things? Different forces are powerful on different scales, and that's why something that can move entire galaxies is not going to affect things within those galaxies. --Captain Ref Desk (talk) 20:04, 21 March 2008 (UTC)[reply]
1. If the answer is to do with the big bang 'throwing out matter spherically' then one would expect a universe that has the galaxies concentrated at the edge.
2. Another answer could be that the theory is simply inconsistent.
3. Another answer would be that everything should be explanding equally but the forces between 'bonded' matter (ie the icebergs) stops them doing so (as you say). ie/eg gravity affects the expansion - regions with more gravity expand less. ie the gravity forces between galaxys is very weak.
3a. Think in terms of weak springs and strong springs holding everything together - the strong springs are under tension but do not expand much eg molecules/icebergs. The weak springs are under tension but stretch lots - hence galaxies are far apart - In this case it helps if you assume that the expansion of space is accelerating.87.102.16.238 (talk) 20:14, 21 March 2008 (UTC)[reply]
3a (sub) You don't need to throw away standard euclidean space in 3a. This model works using simple concepts such as spring constant - which can be directly related to electromagnetic and gravitational forces. The only requirement for success is that the lengths are increasing and the rate of increase is increasing eg the metric is accelerating as it increases in length eg L=lenght dL/dt>0 AND d2L/dt2>0. I can't emphasise this enough!
[edit conflict] You may be right, Captain, based on this, but if so I don't understand why, because metric expansion isn't a force. (Is it?) And excellent question, IBangMyHead. --Allen (talk) 20:21, 21 March 2008 (UTC)[reply]
Yes - metric expansion can cause a force - see above comment 3a, 3a(sub) - that would go some way to explaining things - no idea if that is really true.87.102.16.238 (talk) 20:23, 21 March 2008 (UTC)[reply]
It looks like the simplest explanation for this phenonama would be that all things in the universe experience an additional force (additional to the ones you know eg gravity,electro ) that is universally repulsive. Only strongly bonded things can resist this force. What is the name if this force?87.102.16.238 (talk) 20:43, 21 March 2008 (UTC)[reply]
There's no reason to think it's not a force just because nobody else has stated it is or isn't - it acts like a force therefor it is a _____.87.102.16.238 (talk) 20:45, 21 March 2008 (UTC)[reply]
Here is some perfectly relevant discussion from Argonne National Lab, inspired by our own article. Apparently even (non-cosmologist) physicists can be confused by this question. --Allen (talk) 21:14, 21 March 2008 (UTC)[reply]
And Michael Pierce seems to agree with you, 87, although he doesn't name the force resulting from expansion. --Allen (talk) 21:20, 21 March 2008 (UTC)[reply]
It's interesting that both my question and the one raised on the Argonne Lab page were inspired by the same Wikipedia article, and by the comparison of the "raisin" and "rubber" models in particular. Captain Ref Desk's point that "The structure of atoms themselves are regulated by nuclear force, which metric expansion of space couldn't affect even if it wanted to, and the structural integrity of molecules is going to preserved by the electromagetic force" is one that I had alluded to in my question - one of the things that had caused me to ask the question was that I had in fact considered this, and on thinking about it, it seemed that the internal structure of a basic molecules could be computed (e.g. internal distances for a stable molecule) so any separation caused by inflationary pressure would end up being reversed by the internal forces anyway (and in practice the "spring" would not expand and then contract - merely resist) but this seemed to suggest the idea itself that there is an internal "reactive force" counteracting an "inflationary force" - the fact that distant galaxies have no equivalent possibility for a "reactive force" could then explain why they were affected by the inflation. But as I couldn't find anything written about this force - or at least analogy to a force - I came to the refdesk. In particular I'm not sure just how sound the analogy (or literal truth of inflationary "forcehood") is, and what consequences it has. For example, if you view it as a force, does it mean that the "springs" of atomic bonds are somehow in (very slightly) more "tension" as a result? And if it is a force, then does this have any energy implications (not all forces have an energy associated with them, but some obviously do)?
One thing that was maybe too subtle for both me and Captain Ref Desk is the comment in the Argonne link that "If everything (and I mean everything) were changing with time, such that the universe, the yard-stick, and the relative force strengths (the coupling strength) were all changing in the same way and proportionally, then there would be no way to measure such a change" which is rather insightful. You obviously can't make the universe twice as big in any meaningful way by getting ISO to define "1 new metre = 2 old metres" because, for example, formulae for gravitational attraction would simply change from Fold = GMm/r2 (r measured in "old metres") to Fnew = GMm/R2 (R measured in "new metres" so R = 2r; the two forces would actually have different but convertible units) and there would be no observable difference. (Mental note: when metric conversion finally happens, switching from yards to metres will not increase the size of the universe by around 10% and my waist will not be 10% larger...) When we say "the metric changes" we obviously don't mean it in this naive way. Stretching my intuitive understanding a little bit, is a reason that we might look at inflationary pressure as a force, that the formulae for e.g. inverse square attraction/repulsion somehow don't change their distances, but the distances themselves should change - and if the distance doesn't change, it is apparently because a reactive force has constrained it from doing so? 87.113.64.15 (talk) 22:11, 21 March 2008 (UTC)[reply]
Note to 87.113 - by the time the USA finally switches to the metric system, I pretty much guarantee that your waist -will- be 10% larger :) Franamax (talk) 00:55, 22 March 2008 (UTC)[reply]

There is nothing pushing the galaxies apart (ignoring for the moment the cosmological constant, about which more later). The galaxies are moving apart because of inertia. They were moving apart in the past, and nothing has stopped them—gravity has only slowed them down a bit, and the other forces don't operate between galaxies at all. The galaxies have been moving apart since they formed, because they condensed from matter that was moving apart, and they acquired the average velocity of that matter. But they did condense; that means that local gravitation overcame the initial separating velocities, just like the gravity between the earth and an upward-thrown ball overcomes their initial separation velocity. Once the initial separation has stopped, that's that. There's nothing trying to make the ball fly away from the earth again, it just happened to be flying away to begin with in the unexplained initial conditions of the problem.

Aside from making galaxies and clusters condense out of the primordial Hubble flow, gravity also makes the Hubble expansion as a whole slow down. Absent a cosmological constant, the recession speed of a typical pair of galaxies can only decrease with time. But the more they recede from each other the weaker the attraction between them becomes. If the matter density of the universe is high enough (above the critical density), then the attraction wins out and the universe recollapses (i.e. the ball falls back to earth). If it's not high enough then the galaxies disperse too fast for gravity to pull them back together (i.e. the ball exceeded the escape velocity). In the latter case the recession velocities approach a limiting value and the galaxies simply recede linearly away from each other forever, with essentially no further gravitational interaction since they're so far apart.

Okay, now add a cosmological constant to this. Probably the best way to think of the cosmological constant is as a correction to the gravitational force. Along with an attraction proportional to m/r², you also have a repulsion proportional to r. At small separations the attaction wins out; at larger separations the attraction decreases and the repulsion increases until eventually there's a crossover point and the repulsion dominates. With regard to the overall motion of the universe the situation is not much different from before; there's still a critical density below which the attraction wins and above which it doesn't, except that now if the attraction doesn't win, the far future motion is exponential instead of linear (because the solution to the differential equation is exponential). But again this only affects large clusters of matter which are still moving in the aggregate with the Hubble flow. For things which have condensed out of the Hubble flow, like our galaxy, the short-range attractive force dominates to such an extent that you can't even detect the repulsive term with the most delicate experiments. About the only visible consequence of the runaway separation is that you lose sight of other galaxies (and the CMBR). This eventually happens even without the cosmological constant, but it takes a lot longer. Other than that, life continues as normal until heat death (sorry, I know it's a downer). One caveat is that it's not clear yet whether the dark energy actually behaves like a cosmological constant. If it doesn't, there are other scenarios like the big rip. -- BenRG (talk) 00:19, 22 March 2008 (UTC)[reply]

Excellent answer, BenRG; thank you. But it leaves me confused about where "metric expansion" fits into the picture. I had the impression that for most of the 20th century, metric expansion was accepted based on Hubble's observations, but that the cosmological constant was rejected until the 1990s. So metric expansion != cosmological constant, therefore your first two paragraphs must be dealing with metric expansion. But you make it sound like a simple matter of galaxies flying apart in a Newtonian sort of way, whereas metric expansion is supposed to be spookier than that, as in space itself expanding... whoa! Is there a straightforward way to relieve some of my confusion? (I'm mindful that if one keeps asking physics people seemingly logical cosmological questions, one will eventually be told, "Sorry, beyond this point there's no substitute for learning the math.") --Allen (talk) 01:14, 22 March 2008 (UTC)[reply]

"He's got balls..."

Is there any real correlation between human testicle size and the propensity for bravery and courage? --81.77.147.21 (talk) 22:27, 21 March 2008 (UTC)[reply]

The verifiability and falsifiability of a claim would be in question. Mac Davis (talk) 00:28, 22 March 2008 (UTC)[reply]
It's fairly well established that high testosterone levels cause aggressive and risk-taking behavior. However, the link between testicle size and high testosterone level is less certain. In the case of athletes taking steroids, for example, they often have high testosterone levels and small testicles. StuRat (talk) 03:01, 22 March 2008 (UTC)[reply]
For what it's worth, our article on testosterone claims that it's low levels that cause aggression. --Milkbreath (talk) 03:23, 22 March 2008 (UTC)[reply]
The IP who inserted that information (without references) in October has a string of vandalism warnings around the same time and was then blocked. I would suggest that can safely be put down to vandalism and 82.42.171.4's other edits should be reviewed at as well. SpinningSpark 08:49, 22 March 2008 (UTC)[reply]
Of course there is - but only in men. silly.87.102.16.238 (talk) 11:41, 22 March 2008 (UTC)[reply]

dimethyl sulfone decomposition

Does dimethyl-sulfone (MSM) decompose in household storage, or is it stable? Would it be an oxidation or reduction reaction?

If you keep it in the bottle it should be fine. It should stay that way for years. Mac Davis (talk) 23:56, 21 March 2008 (UTC)[reply]
Are you asking again with different words? We don't offer advice on how things work inside your body. Even if it's not a medical compound, really, ask a pharmacist. Franamax (talk) 00:39, 22 March 2008 (UTC)[reply]

It is kept in the bottle in a cupboard and opened daily, with a bit spooned out. The spoon is always clean and dry, but the powder balls up a little due to attracting moisture in the air. Someone reported the same "balling up", but they also said that dimethyl-sulfone does not decompose significantly over time.

The question is straightforward and not about physiological application. I am sure the WP community has thousands of participants of any given background, but anyone knowledgeable about the stability and/or reactivity of dimethyl-sulfone at room temperature can address my question. Thank you.

dimethyl sulfone should keep for a long time when stored correctly eg in the dark, stopper on. Just how long I don't have the answer. But it is in general stable (like sugar etc)87.102.16.238 (talk) 11:50, 22 March 2008 (UTC)[reply]
Yeah make sure it is in the dark. Double bonds never like UV light.--Shniken1 (talk) 12:10, 22 March 2008 (UTC)[reply]

March 22

Automatic crossbow

Has there ever been such a device as a fully automatic crossbow? If not, is there any technical reason why not? --Kurt Shaped Box (talk) 01:14, 22 March 2008 (UTC)[reply]

Well we have an article repeating crossbow. Also take a look at Zhuge Liang#Legacy the ancient chinese supposed inventor of these.
Also this thread [18] describes unmanned automatic crossbows mounted on chariots driven by the turning chariot wheels and flung at the enemy by setting fire to the horses tails. No idea how reliable this source is, but it conjures up a wonderful picture. SpinningSpark 01:36, 22 March 2008 (UTC)[reply]
I'll test this next time I go by a stable, assuming I can find a used chariot. But what happens when the horses turn around and come running back? Maybe that's why this method of warfare didn't catch on? :) Franamax (talk) 12:27, 22 March 2008 (UTC)[reply]
Just be thankful that you weren't fighting alongside war pigs (if they ever existed). --Kurt Shaped Box (talk) 13:17, 22 March 2008 (UTC)[reply]
Why would they come running back? If someone set your arse on fire would you go back there? SpinningSpark 13:29, 22 March 2008 (UTC)[reply]
If I was on fire, I probably wouldn't be paying that much attention to where I was running. --Kurt Shaped Box (talk) 13:37, 22 March 2008 (UTC)[reply]
(e/c) Ahh, war pigs, I always wondered what Black Sabbath was going on about! SS, why would the horses keep running in a straight line? If your butt's on fire, do you keep track of directions? (Don't try this at home kids) What I was thinking of though is that they might run back towards some of the unburnt horses they were just hanging around with, horses tend to do that. And they would have kept at least one horse for the general to ride afterwards, right? Who wants to ride a smelly burnt horse? :) Franamax (talk) 13:45, 22 March 2008 (UTC)[reply]

water heater is making a whining sound

I live in a two story house. Our water heater is making a whining sound that can be heard all through the house. I'm afraid that a pipe or the water heater is going to blow and cause a terrible problem. Do you know what could be making the whining sound that has never been heard in our house before? What can we do about it?

Gas? Fueloil? Electric? Immersion? A little more detail would help. --BozMo talk 06:55, 22 March 2008 (UTC)[reply]
It sounds like you need a plumber to not only examine the water heater, butt crack this problem wide open before a serious leak occurs. StuRat (talk) 11:41, 22 March 2008 (UTC)[reply]
Googling on "water heater" whining provides many useful links. I have heard that they can make noises when there is a layer of deposits in the bottom in which water can become superheated and be forced through the deposits. It is possible that the pressure relief valve is starting to leak. I would warn you that the water heater is nothing to mess around with; when things go wrong they can go wrong big. Here is a video from Mythbusters that proves that. --Milkbreath (talk) 13:35, 22 March 2008 (UTC)[reply]
I'd amend that to "the water heater is nothing to mess around with if you don't know what you're doing". In my opinion that Mythbusters video is (like much popular entertainment) somewhat alarmist. The fact that you never hear about water heaters exploding like that in real life is good proof that, virtually all of the time, the designed-in safety mechanisms work as they're supposed to. I'm not saying it never happens, but anyone who tampers with or disables a safety device such as a T&P valve displays criminal negligence and stupidity. —Steve Summit (talk) 14:05, 22 March 2008 (UTC)[reply]
Addendum: Never? Well, hardly ever. Yikes. —scs 14:10, 22 March 2008 (UTC)[reply]
Yes, the Mythbusters are not a reliable primary source as a rule, but I watched that one, and we're seeing the true consequences of defeating the safety devices, turning the thermostat up, and then simply turning the thing on and waiting. No extra explosives or other heroic measures were used for dramatic effect. Both they and I were astonished at the violence of the event. Apparently, such explosions were commonplace before the introduction of the safety valve. --Milkbreath (talk) 14:41, 22 March 2008 (UTC)[reply]
Right. But the nice thing is, today, simultaneous failures of both the thermostat and the safety valve are exceedingly rare. I'd wager that all such failures are due to capping off of the safety valve (as in the referenced Seattle P-I article, presumably because the valve was leaking). —Steve Summit (talk) 15:16, 22 March 2008 (UTC)[reply]

Best Food

If a human had to survive on just water and one other substance, what would that substance be ?

In order to survive as long as possible, I think it would be sodium chloride. Without it he would die pretty soon from water poisoning. Icek (talk) 15:31, 22 March 2008 (UTC)[reply]
Does cornish pasty count as a substance? Or how about earth (soil) - with earth and water you could grow your own food..87.102.16.238 (talk) 16:22, 22 March 2008 (UTC)[reply]

Garlic for treatment of wounds

Hello, there is hearsay about usage of garlic for the treatment of wounds up to the time of WWI. For the respective WP articles, I'd need at least one account that it was official treatment. Google was no use, neither the Project Gutenberg full text search.

Do you know of a passage in a book (even fiction) where garlic was used on a wound, or its usage was recommended by an official about 100 years ago? --85.179.13.254 (talk) 17:10, 22 March 2008 (UTC)[reply]

Genetic mutation resulting in deformities of the offspring

  1. In sexual reproduction, i.e., fertilization of egg by sperm, does mutation of the egg or sperm producing organs, say by means of radiation, always result in the failure of the organs to produce viable egg or sperm, i.e., egg or sperm that can result in a fertilized egg? In other words, are deformities in an offspring due to mutated egg or sperm producing organs?
  2. If mutated egg or sperm producing organs are not responsible for deformities in an offspring then is mutation of the egg or sperm, say by irradiation, after they are produced and prior to fertilization responsible for deformities in the offspring?
  3. If deformities do not result from mutated egg or sperm occurring after they are produced, say by irradiation, then are deformities in the offspring the result of a faulty union of egg and sperm?
  4. If faulty union of egg and sperm are not responsible for deformities in the offspring then is the genetic mechanism by which deformities occur in the offspring the result of mutation of the fertilized egg after it is fertilized, by say irradiation of the fertilized egg? 71.100.1.14 (talk) 17:18, 22 March 2008 (UTC)[reply]